Sunteți pe pagina 1din 56

www.gradeup.

co

1
www.gradeup.co

SBI PO 2019: Free E-Book

Table of Content
I. Introduction
II. SBI PO Exam Details
A. Changes introduced in SBI PO Notification
B. SBI PO 2019 - List of Important Dates
C. SBI PO Job Profile
D. SBI PO Salary
E. SBI PO Career Growth
III. SBI PO - Exam Pattern and Syllabus
A. Preliminary Exam Pattern
B. Main Exam Pattern
C. SBI PO Exam Syllabus (Prelims and Main)
IV. SBI PO Previous Year Cut-off (Prelims as well as Main)
A. SBI PO Preliminary Exam Cut-off (2018, 2017 & 2016)
B. SBI PO Main Exam Cut-off (2018, 2017 & 2016)
C. SBI PO Final Cut-off (2018, 2017 & 2016)
V. Study Material for SBI PO
VI. Strategy to crack SBI PO Exam 2019
VII. Previous Year Question Papers of SBI PO
VIII. Free Mock Test for SBI PO 2019

Introduction
State Bank of India has released the recruitment notification for SBI PO Exam 2019.
SBI PO is one of the most sought-after exam of the country and every year almost 10 lakh students appear for
this exam. This year, SBI has released the notification for 2000 vacancies and students, approximately 10
times the number of total vacancies, will be shortlisted for the Main exam.

To ramp up your preparations for SBI PO Exam, we are providing you this handbook that will cover all the
major aspects of this exam so that you do not miss out anything important. Prepare well.

Happy Reading...!!

SBI PO Exam Details


This year, SBI PO notification has been released for 2000 vacancies. SBI has also introduced some important
changes in the pattern of PO exam this year.
A. Changes introduced in SBI PO 2018
Just like last year, this time SBI PO recruitment has come up with the following major changes:
1. The sectional marking scheme has been defined this time for the SBI PO Mains 2019 exam
2. Sectional time limit has been introduced for each section in SBI PO Prelims exam
3. There is no sectional cut off for SBI PO Preliminary as well as Mains exams. The final selection of the
candidates will be based on their performance in each phase of the exam
B. SBI PO 2018 - List of Important Dates
Here is the complete exam calendar for the entire recruitment process of SBI PO 2018:

2
www.gradeup.co

Download of Call Letters: Preliminary Exam 3rd Week of My 2019 onwards

Online Examination - Preliminary 8th, 9th, 15th and 16th June 2018

Result of Preliminary exam 1st Week of July 2019

Download of Call letters for Mains 2nd Week of July 2019

Mains Online Examination 20th July 2019

Result of Mains Exam 3rd Week of August 2019

Download of Call letters for Interview 4th Week of August 2019

Conduct of Group Exercises & Interview September 2019

Declaration of Final Result 2nd week of October 2019

You can also go through the link below to view the official notification of SBI PO 2019 –

Official Recruitment Notice of SBI PO 2019!

C. SBI PO Job Profile


• It is essential to understand what kind of tasks one might have to undertake after getting selected as a
probationary officer in SBI.
• The candidates after selection undergo a probation period of 2 years.
• Candidates undergo training in this period to understand the various aspects of banking procedures in a
practical way.
D. SBI PO Salary 2019
One of the most asked questions among students is the in-hand salary that they would receive as a
probationary officer in SBI. The basic grade pay is Rs. 27620/- which is accompanied by 4 increments. Apart
from this, the salary component involves other Allowances which is as follows. For more details, you can check
the SBI PO Salary.

Rs.27,620/- (with 4 advance increments)


SBI PO Basic Pay
(in the scale of 23700-980/7-30560-1145/2-32850-1310/7-42020)
This is revised every quarter and is calculated based on CPI (Consumer Price
Dearness Allowance
Index). Currently, DA is 46.9% of the basic pay.
City Compensatory
This varies with the place of posting at 4% or 3%.
Allowance (CCA)
SBI PO may take a house on lease, the amount of which varies with posting
Leased House
between a minimum of. Rs. 8000/- (in rural areas) and a maximum of Rs.
Accommodation
29500/- (in Mumbai).
Furniture Allowance Rs. 120000/- for the purchase of furniture
Medical Insurance 100% medical coverage for SBI employees and 75% for their family members
Travelling Allowance For official travels, reimbursement of AC 2-tier fare is provided
50Lts (For 2 wheelers) Or 55Lts (For 4 Wheeler) Or Approx. Rs. 1100-1250 if you
Petrol
don't have 2/4 wheeler (conveyance allowance).

3
www.gradeup.co

E. Promotion and Career Growth


The career growth of an SBI PO is mentioned below-
• Probationary Officer (Assistant Manager, JMGS - I)
• Deputy Manager (MMGS)
• Manager (MMGS)
• Chief Manager (MMGS)
• Assistant General Manager (SMGS)
• Deputy General Manager (TEGS)
• Chief General Manager (TEGS)
• General Manager (TEGS)
F. SBI PO 2019 Selection Procedure
The Selection Procedure of SBI PO involves the following phases-
• Preliminary Exam (Objective)
• Mains Exam (Objective + Descriptive)
• Group Exercise and Interview

SBI PO - Exam Pattern and Syllabus


SBI PO Exam will be conducted in following 3 phases -
1. Preliminary Exam (Objective)
2. Mains Exam (Objective + Descriptive Exam)
3. Group Exercises and Interview
A. Preliminary Exam Pattern –

Section No. of Questions Maximum Marks Duration


English 30 30 20 Minutes
Reasoning 35 35 20 Minutes
Quantitative Aptitude 35 35 20 Minutes
Total 100 100 60 Minutes

B. Main Exam Pattern -


The Mains exam of SBI PO consists of 4 sections for a total of 200 marks which are as follows:

Name of Tests (Objective) No. of Ques Max Marks Time


Reasoning & Computer Aptitude 45 60 60 Mins
Data Analysis & Interpretation 35 60 45 Mins
General/ Economy/ Banking Awareness 40 40 35 Mins
English Language 35 40 40 Mins
Total 155 200 3 Hours

Descriptive Exam -
The Descriptive Test of 30 minutes duration with two questions for 50 marks will be a Test of English Language
(Letter Writing & Essay).
The marks obtained by the students in the Mains phase (both objective and descriptive) will be added to the
Group Exercise and Interview marks while preparing the final merit list.

4
www.gradeup.co

C. SBI PO Exam Syllabus


One may expect questions from the under-mentioned topics in the SBI PO Exam:
1. Reasoning Ability -
● Puzzle
● Seating Arrangement (Circular/Linear/Square)
● Inequality
● Syllogism (Old and New Pattern)
● Coding - Decoding (Old and New Pattern)
● Blood Relations
● Direction sense
● Order and Ranking
● Series (Numeric/ Alphabet/ Alphanumeric)
The above-mentioned topics have high probability of being asked in the Prelims phase. Apart from these
topics, questions from following topics may also be asked in Main Phase-
● High Level Puzzles
• Input - Output
● Data Sufficiency
● Verbal/ Logical Reasoning -
Statement and Assumption/ Conclusion/ Course of Action/ Argument
Cause and Effect
Deriving Conclusion from Passage
Selection Criteria based
2. Quantitative Aptitude
● Number Series
● Simplification/ Approximation
● Quadratic Equations
● Data Interpretation
● Mathematical Inequality (Quantity based questions)
● Miscellaneous (Age, Mixture Allegation, Partnership, Pipes and Cisterns, Profit Loss, Average, Boats &
Stream, Compound/Simple Interest, Time & Work, Probability etc.)
● For the Main phase of SBI PO, thoroughly prepare Data Interpretation and Data Sufficiency.
3. English Language
● Reading Comprehension
● Cloze Test (New and Old Pattern)
● Para jumbles (New and Old Pattern)
● Spotting Errors (New and Old Pattern)
● Sentence Improvement
● Fill in the Blanks
Apart from the above-mentioned topics, also cover following for the mains phase -
● Paragraph Completion
● Phrase Starter/ Connector based questions
● Synonyms/ Antonyms
4. General/Economy/Banking Awareness
● This would include current affairs-based questions of the last 5-6 months.
● Try to give special focus to the major financial news of the last 5-6 months.
● Also cover the topics of static banking awareness like important financial terms, their full forms etc.
● You may also expect 3-4 questions from static general awareness like important days, national parks
etc.
To know about the weightage of the above-mentioned topics in exam, check the detailed syllabus of SBI PO
2019 here!

5
www.gradeup.co

SBI PO Previous Year Cut-off (Prelims as well as Mains)


Here are the previous year cut-off scores of SBI PO Exam for Prelims as well as Mains exams.
However, please remember that these cut-offs are just for reference purpose. Do not align your preparation
exactly based on these scores.

A. SBI PO Prelims Cut Off 2018

Minimum Qualifying Marks (Cutoff Score)


Sections/ Exam Total Marks
SC ST OBC GENERAL OH VI HI
Prelims Exam 100 49 43 54.25 56.75 45.25 49 14.75

B. SBI PO Final Cutoff 2018

Total Minimum Qualifying Marks (Cutoff Score)


Sections/ Exam
Marks SC ST OBC GENERAL OH VI HI
Minimum marks to be secured, to be
shortlisted for GE & PI Round 250 77.13 75.01 86.42 93.10 75.03 88.91 75.16
(Objective + Descriptive)
Group Exercise + Interview 50 18 18 18 20 18 18 18
Total Cutoff Marks after Normalisation 100 43.99 39.87 46.26 50.79 43.04 47.09 32.69

C. SBI PO Preliminary Exam Cut-off (2017 & 2016)

Cutoff Scores of SBI PO Pre


Cutoff Scores of SBI PO Pre 2017
2016
Total
Name of Subject
Marks
SC/ ST / OBC /VH /OH SC/ ST / OBC /VH /OH
General General
/ HI / HI

English Language 30 5.75 8.75 6.00 8.75

Quantitative
35 7.00 10.25 4.25 7.25
Aptitude

Reasoning Ability 35 4.00 6.50 4.25 7.00

Overall
SC ST OBC GEN OH VI HI
Cut-off

2017 43.25 31.25 48.25 51.50 38.75 42.50 18.50

2016 38.75 29.25 44.50 47.50 36.00 37.00 15.75

6
www.gradeup.co

B. SBI PO Main Exam Cut-off (2017 & 2016)

SBI removed the sectional cutoff in the mains exam of 2017. Here are the overall cutoff scores of SBI PO Main
exams of 2017 and 2016:

Maximum
Exams Year SC ST OBC GEN OH VI HI
Marks

Online 2017 40 40 40 50 40 40 40
Objective 200
Test 2016 60 60 60 70 60 60 60

2017 17.5 17.5 17.5 20 17.5 17.5 17.5


Descriptive
50
Test
2016 17.5 17.5 17.5 20 17.5 17.5 17.5

Minimum
marks to be
secured for 2017 62.5 62.5 79.5 89.25 62.5 88.09 62.5
GD &
250
Interview
(Objective +
Descriptive 2016 78 78.75 87.25 90 82.75 79.5 79.25
test)

2017 18 18 18 20 18 18 18
GD +
50
Interview
2016 18 18 18 20 18 18 18

C. SBI PO Final Cut-off Marks (2017 & 2016)


Total Marks secured by last ranked selected candidates (Out 0f 100) = Written Test (Normalized to 75) + GD &
Interview (Normalized to 25). The table below consists of the final cutoff marks of SBI PO (out of 100) for the
year 2017 and 2016.

Year SC ST OBC GEN OH VI HI

2017 38.95 28.88 40.83 46.59 37.24 46.11 32.92

2016 34.25 34.73 44.43 49.75 38.45 43.08 40.78

Study Material for SBI PO 2019


Here is the list of section-wise study material, which will help you prepare for the SBI PO exam in a better
way:
1. Quantitative Aptitude
Name of the Topics Quizzes
● Number Series Notes
● Simplification/ Approximation Notes
● Topic-wise Quantitative Aptitude
● Quadratic Equation Notes
Quizzes
● Data Interpretation Notes
(Basics, Pie chart, Bar chart, Tabulation, Missing DI, Caselet DI,

7
www.gradeup.co

Line chart, Mixed DI


● Simple & Compound Interest Notes
● Mixture & Alligation Notes
● Partnership Notes
● Pipes & Cistern Notes
● Notes of Problem on Ages
● Profit, Loss & Discount Notes
● Time & Work Notes
● Probability Notes
Apart from these notes, also check important tips to improve your calculation speed.
2. Reasoning Ability
Name of the Topics Quizzes
● Inequality Notes
● Syllogism Notes (New Pattern & Old)
● Coding-Decoding Notes (New Pattern & Old)
● Puzzle Notes
● Seating Arrangement Notes
● Topic-wise Reasoning Ability Quizzes
● Input-Output Notes
● Data Sufficiency Notes
● Verbal Reasoning notes
● Blood Relation Notes
● Distance & Direction Notes
3. English Language
Name of the Topics Quizzes
● Reading Comprehension Notes
● Cloze Test Notes
● Spotting Error Notes ● Topic-wise English Language Quizzes
● Sentence Correction Notes ● Vocabulary Quizzes
● Fill in the Blank Notes ● Grammar Quizzes
● Para jumbles Notes
● Sentence Connector Notes
Apart from these notes, you must also go through following:
● Daily Wordlist for Vocabulary
● Free PDF of Grammar Rules
● Notes on Grammar Rules
● Important Idioms & Phrases
4. General Awareness
Name of the Topics Quizzes
● Daily Current Affairs
● Free PDFs of Weekly One Liners
● Banking Awareness Quiz
● Free PDFs of Monthly Current Affairs
● Current Affairs Quiz
● Banking Awareness Notes
● Static GK Quiz
● Current Affairs related to Banking
● Static General Awareness Notes
● Important Current Affairs Notes

8
www.gradeup.co

Strategy to crack SBI PO Exam 2019


One needs to put in extra efforts in the preparation of SBI PO exam given its level of competition. The prelims
exam of SBI PO 2018 will commence from 8th June and one must utilise each and every day before the exam,
to its fullest, in order to be shortlisted for the Mains phase.
Look at the in-depth analysis of its previous year's exam patterns just to get an idea about the probable
expectations in the paper this year. Let's discuss it year wise & subject wise. We will be sharing the details of
each subject for all the three years i.e, 2018, 2017 & 2016.
Major Changes in Exam as compared to Last Year

Subject/year 2016 2017 2018


• Level: Moderate
• Highlights: The pattern
of English Language got
• Level: Easy to Moderate changed. There were
• Level: Easy
• Highlights: RC was of the questions from many new
• Highlights: English
moderate level and not that topics like Sentence
was quite simple
easy. Rearrangement, Match the
English with typical
• Important Changes: There column, Finding the correct
Language sentence
were no questions from sentence etc in the different
rearrangement.
Parajumbles or Fillers this shifts.
Rest questions were
year, Cloze test was based on • Important Changes:
easy.
the new pattern this year. Different topics were asked in
different shifts.
*Sectional timing of 20 mins.
was introduced.
• Level: Moderate
• Highlights: Puzzles were of
• Level: Easy to Moderate
a different type. For example,
• Highlights: Maximum
• Level: Easy to in the linear arrangement,
questions(23) were from
Moderate the number of persons were
Reasoning puzzles and seating
• Highlights: not specified.
Ability arrangements.
Easiest and most • Important
• Important Changes: No
scoring section. Changes: Syllogism was not
major change in exam
asked this year.
pattern.
*Sectional timing of 20 mins.
was introduced
• Level: Easy to Moderate
• Level: Moderate
• Highlights: DI questions and
• Highlights: Data sufficiency
Approx. questions were
• Level: Easy and Quantity based questions
easiest. Instead of
• Highlights: were asked.
simplification questions,
Quantitative Arithmetic questions • Important Changes: Wrong
Quadratic Equation were
Aptitude were of a moderate number series, quantity
asked, the rest paper
level and rest was based questions were asked
was almost the same
easy in the prelims exam.
• Important Changes: No
*Sectional timing of 20 mins.
major change in the exam
was introduced
pattern

9
www.gradeup.co

English Language Exam Analysis of 2018/2017/2016


2018

2017

10
www.gradeup.co

2016

11
www.gradeup.co

Quantitative Aptitude Exam Analysis of 2018/2017/2016

2018

2017

12
www.gradeup.co

2016

Reasoning Ability Exam Analysis of 2018/2017/2016

2018

13
www.gradeup.co

2017

2016

Here are some important preparation tips for SBI PO 2019:

14
www.gradeup.co

1. Quantitative Aptitude Tips


● Thoroughly prepare and practice data interpretation-based questions.
For this, start with the basic chapters like Ratios, Percentages (calculation of % increase/ decrease),
averages, Profit/ Loss etc, since questions of data interpretation are based on these topics.
● Practice at least 5-6 sets of DI questions on a regular basis.
● Questions of Simplification/ Approximation are based on basic BODMAS rules and can be mastered with
regular practice. So, devote at least 15-20 minutes to the practice of Simplification/ Approximation
questions.
● For Number Series, first identify the common patterns which are generally asked in the exams like,
(a) basic arithmetic calculation based
(b) Squares/ Cubes based
(c) Square root/ Cube root based
(d) Operations in double step
These patterns usually come up in a number of bank exams and with thorough practice, identifying
them in the questions will become easy.
● Questions of Quadratic Equation are usually factorization based.
● Understand the concepts of the miscellaneous arithmetic topics, common formulas used in the
questions and then practice questions based on them.
2. Reasoning Ability Tips
● A thorough practice of seating arrangement and puzzle-based questions is a must. But before that,
make sure that you have a proper understanding of the approach used to solve such questions.
● Inequality is one of the easiest topic of Reasoning section and proper practice of this topic can easily
fetch you 5 marks in the exam.
● The remaining topics of reasoning section, like Syllogism, Coding-Decoding, Blood relation, Direction
sense etc although carry less weightage but are doable and hence, must be practiced properly.
3. English Language Tips
● Reading Comprehension in SBI PO exam is expected to be economy/ social/ environmental issue
related.
Since sectional time of 20 minutes is available, you can easily attempt the RC question in English. So,
work on your reading skills
● Read the newspaper editorials of The Economist properly and work on your vocabulary.
This will also help you in Cloze test-based questions.
● For Sentence Improvement and Spotting Errors, understand the basic grammar rules.
● Regular practice is very important for the questions of English section. Once you have attempted the
questions, do check their solutions to understand the mistakes (if any) which you committed or
alternate methods to solve the question.
You can also check important tips for the preparation of SBI PO Exam from the article already shared by us.
We have also started a daily Study Plan for SBI PO 2019. You can follow it from the link shared below for
better preparation of this exam:

Daily Study Plan for SBI PO 2019

Previous Year Question Papers of SBI PO


From the link shared below, you can download the free PDFs of the previous year question papers of SBI PO
(2018, 2017, 2016 & 2015) for both, preliminary and mains phases, along with their detailed solutions.
You can also download gradeup to practice these papers online!

Previous Year Question Papers of SBI PO (Pre & Mains), Download Now!

Free Mock Test for SBI PO 2019

15
www.gradeup.co

Direction (1-3): Read the following placed at the bottom. An equal number of
information carefully to answer the given boxes are kept between T and A as in
questions. between R and U. Box A is pink colored. Only
In a certain code language, Mustard and Magenta colored boxes are
‘need honest people’ is written as ‘me ga pa’. placed between A and P. One of the box
‘hour of need ’ is wriiten as ‘ne me go’. placed is V. Box Y is yellow colored.
‘must have hour’ is written as ‘se ne ge’. 4. How many empty boxes are there in the
1. Which of the following is the code for ‘of’? arrangement?
A. ne B. ge A. 1 B. 2
C. me D. go C. 3 D. more than 3
E. Cannot be determined E. can not be determined
2. What does ‘ge’ stand for? 5. Which of the following boxes is placed
A. hour B. of between S and A?
C. must D. have A. Magenta B. Mustard
E. cannot be determined C. Brown D. Green
3. In the given code language, what may be the E. can not be determined
possible code for ‘ne so pa’? 6. Which among the following boxes is kept
A. good hour people immediately above W?
B. good honest people A. T B. S
C. buses good hour C. V D. Q
D. buses people honest E. None of these
E. None of the above 7. Which one of the following is a Red colored
box?
Direction (4-8): Study the following A. W B. V
information carefully and answer the given C. S D. U
questions: E. can not be determined
A certain number of boxes are kept one above 8. Box R is of which color?
the other in such a way that the box at the A. Red B. Brown
bottom is numbered 1 and the one above it is C. Blue D. Mustard
numbered 2 and so on. Each box is of a E. can not be determined
different color.
More than 1 empty box is placed in the Directions (9-11): Study the following
arrangement. information and answer the given questions-
Box U is not placed at the top. A is placed A, B, C, D, E, F, G and H are 8 family
below T but above S. Brown coloured box is members. B is the only son of C. K’s aunt, E is
placed immediately above the Red coloured married to H. A’s daughter-in-law, D is the
box. Neither S nor R is silver coloured. Box W mother of K. A is married to C. D has no
is not red coloured. Brown box is placed siblings. J is the brother of K
immediately below the box S. Silver coloured 9. How is K related to A?
box is placed at the top. An equal number of A. Nephew B. Niece
boxes are kept between P and Q as there are C. Grandson D. Granddaughter
in between R and S. More than 7 boxes can E. Cannot be determined
be kept between Q and silver coloured box. 10. How many female members are there in the
Box Q is green coloured. There are 5 boxes family?
between P and the silver coloured box. Box T A. 3 B. 5
is white colored. Blue coloured box is placed C. 4 D. 6
above Brown colored box. The box which is of E. Cannot be determined
white color is placed between R and the 11. How is H related to A?
topmost box. Box P is placed immediately A. Son B. Daughter
above the Red colored box. Boxes that can be C. Daughter-in-law D. Son-in- law
placed above P are 1 more than boxes that E. None of the given options
can be placed below P. Yellow colored box is

16
www.gradeup.co

12. Direction: The question below consists of a B. Data in Statement II alone are sufficient to
question and two statements numbered I and answer the question, while the data in
II given below it. You have to decide whether Statement I alone are not sufficient to answer
the data provided in the statements are the question.
sufficient to answer the question. Read both C. Data either in Statement I alone or in
the statements and give answer.
Statement II alone are sufficient to answer
Which direction is Shilpa facing?
the question.
I. If Meeta who is currently facing south turns
90 ° towards her left, she would face the D. Data in both the Statements I and II
same direction as Shilpa. together are not sufficient to answer the
II. If Uday who is currently facing north, turns question.
90 ° towards his left, he would face a E. Data in both the Statements I and II
direction just opposite to the direction Shilpa together are necessary to answer the
is facing. question.
A. Statement I and II together are sufficient.
Direction (15-17): Read the following
B. Statement I alone is sufficient.
information carefully to answer the given
C. Statement II alone is sufficient
D. Statement I alone or statement II alone is questions.
sufficient. 7 friends A, B, C, D, E, F and G have different
E. Statement I and II together are not number of houses in a society. No 2 persons
sufficient. have the same number of houses. No one has
13. How is M related to N? more than 22 houses. D has more houses
I. P, who has only two kids, M & N, is the than C but less than G. Only 2 persons have
mother-in-law of Q, who is the sister-in-law of more houses than E. The person having the
N. second highest number of houses has 17
II. R, the sister-in-law of M, is the daughter- houses. The number of houses that G has is
in-law of S, who has only two kids, M & N. multiple of both 3 and 5. C has exactly 9
A. Data in Statement I alone are sufficient to houses but does not have the least number of
answer the question, while the data in
houses. B does not have 17 houses and he
Statement II alone are not sufficient to
has more houses than A.
answer the question.
B. Data in Statement II alone are sufficient to 15. Which of the following has 16 houses?
answer the question, while the data in A. A B. B
Statement I alone are not sufficient to answer C. D D. E
the question. E. F
C. Data either in Statement I alone or in 16. Which one of the following has the least
Statement II alone are sufficient to answer number of houses?
the question. A. A B. E
D. Data in both the Statements I and II C. D D. F
together are not sufficient to answer the E. cannot be determined
question. 17. Who among the following has the third lowest
E. Data in both the Statements I and II number of houses?
together are necessary to answer the
A. A B. B
question.
C. C D. D
14. Which of the following will indicate color of
clear sky in a coding system? E. F
I. ‘Indigo’ means ‘Grey’, ‘Grey’ means ‘Black’, Directions (18-19): In each of the following
Black’ means ‘Blue’ in that system. questions, assuming the given statements to
II. ‘Black’ means ‘Blue’, ‘Blue’ means be true, find out which of the following
‘Orange’; Orange’ means ‘Green’ in that
conclusions given below is/are definitely true.
system.
18. Statements:
A. Data in Statement I alone are sufficient to
answer the question, while the data in A ≤ B, B > C, C ≤ A, D < A, A = E
Statement II alone are not sufficient to Conclusion:
answer the question. a) D > B
b) E > B

17
www.gradeup.co

A. only conclusion (a) is true. 24. Which of the following statement(s) is/are
B. only conclusion (b) is true. definitely true?
C. either conclusion (a) or (b) is true. A. P sits second from the right end of the
D. neither conclusion (a) nor (b) is true. row.
E. both conclusion (a) and (b) are true. B. More than 2 persons are sitting to the left
19. Statements: 1 ≥ 2, 3 ≤ 2, 6 = 1, 3 = 4, 4 > of V.
5 C. Z sits immediate right of Q.
Conclusion: a) 2 ≥ 4 D. O sits at one of the extreme ends of the
b) 5 < 2 row.
A. only conclusion (a) is true. E. None of these
B. only conclusion (b) is true. 25. How many such pairs of letters are there in
C. either conclusion (a) or (b) is true. the word INSTRUCTION which have as many
D. neither conclusion (a) nor (b) is true. letters between them in the word as in the
E. both conclusion (a) and (b) are true. English alphabet?
Direction (20-24): Study the following A. 0 B. 1
information carefully and answer the given C. 2 D. 3
questions: E. None of these
A certain number of persons are sitting in a Direction (26-28): Study the following
row facing north equidistant from each other. information carefully and answer the
Only 1 person sits between T and Q, who does questions given below.
not sit adjacent to X. S sits to the right of Y. A number arrangement machine when given a
Only 4 persons sit between W and O, who sits particular input rearranges it following a
to the right of Q. T sits adjacent to X, who sits particular rule. The following is the illustration
third from left end of the row. The number of of the input and the steps of arrangement.
persons who sit on the right side of Q is 1 Input: 12 17 14 23 22 19 25 29
more than the number of persons sitting on Step I: 2 7 4 6 4 9 10 18
the left side of him. Y is not an immediate Step II: 4 9 6 8 6 11 12 20
neighbour of X. Only 2 persons sit between W Step III: 12 27 18 24 18 33 36 60
and U. There are more than 2 persons Step IV: 21 72 81 42 81 33 63 6
between Z and V, who sits to the left of Z. Step V: 19 70 79 40 79 31 61 4
Only 4 persons sit between T and R. Q sits to 26. What will be the fifth step of the following
the left of W and V sits to the left of X, who input?
does not sit adjacent to S. Z sits to the Input: 52 71 43 39 47 36 17 19
immediate right of T. A. 12 9 14 29 30 20 9 11
20. How many persons are sitting in the row? B. 61 70 22 76 7 4 70 31
A. 10 B. 11 C. 36 27 42 87 90 60 27 33
C. 12 D. 13 D. 52 71 43 39 47 36 17 19
E. cannot be determined E. None of these
21. How many persons are sitting between Q and 27. What will be the fourth step of the following
W? input?
A. None B. 1 Input: 9 15 7 6 13 22 12?
C. 2 D. 3 A. 33 21 27 24 15 18 12
E. More than 3 B. 9 15 7 6 13 22 12
22. Who is sitting third to the left of the one who C. 11 7 9 8 5 6 4
is sitting second to the right of U? D. 33 12 72 42 51 81 21
A. R B. S E. None of these
C. Y D. O 28. If the first step of the given input is 4 3 12 24
E. None of these 8 21 14 16 what will be the step V for the
23. What is the position of R with respect to S? input?
A. Immediate right A. 81 51 24 87 3 96 84 45
B. Immediate left B. 18 15 42 78 30 69 48 54
C. second to the left C. 6 5 14 26 10 23 16 18
D. second to the right D. 17 15 5 11 8 9 13 10
E. None of these E. None of these

18
www.gradeup.co

Directions (29-33): Study the following 32. Which of the following statement(s) is true
information carefully and answer the given with respect to the team which plays
questions: Basketball?
In a sports meet, different teams – Black, A. Basketball is played on Monday.
Blue, Brown, Green, Mustard, Orange, Pink, B. Only 3 teams play between Volleyball and
Purple, Red, Violet, White and Yellow play Cricket teams.
different games – Archery, Badminton, C. Team orange plays immediately before
Basketball, Cricket, Javelin Throw, Kabaddi, volleyball team.
Kho Kho, Long jump, Ludo, Table tennis, D. There are 2 games which are played
Tambola and Volleyball in different time slots between Basketball and Tambola.
– 8:00am, 12:00pm and 3:00pm on different E. All the given statements are true.
days of a week – Monday, Wednesday, Friday 33. Team pink plays on which day and in which
and Sunday. slot?
Kabaddi is played before Volleyball. Team A. Wednesday at 8:00am
pink plays Kho Kho. Tambola is played on the B. Friday at 3:00pm
same day on which Long Jump is organized. C. Monday at 3:00pm
Archery is played by neither Brown nor violet D. Friday at 12:00pm
team. Team purple plays Basketball at 12:00 E. None of these
pm. Only 2 teams play between Green and
white teams. There are 3 teams which play Directions (34-35): Study the following
between Yellow and Green team. Team yellow information and answer the given questions:
does not play Ludo. Badminton is played by Point U is 35m to the west of Point W. W is
Mustard team. Ludo is played in the last slot 20m to the south of H. H is 25m to the east of
of last day of the week. The number of games S. Sarita is standing at Z which is 40m to the
played between Table Tennis and Long jump south of S. She starts walking towards the
is equivalent to the number of games played east and walks for 30m. She takes a left turn
between Badminton and Javelin Throw. Both and stops at K after walking for 20m.
Table tennis and Long jump are played before 34. If she walks for 5m towards the west from her
Cricket. Archery is played immediately before final position, she will be at which point?
Tambola. Team Black plays Table Tennis. A. H B. S
Team Red plays Cricket on Wednesday in the C. W D. U
third slot. The number of teams playing E. None of these
before Long jump is equivalent to the number 35. In which direction is W with respect to K?
of teams playing after Team yellow. Javelin A. North B. South
Throw is played by team blue immediately C. North-west D. South-west
before Ludo. The number of games played E. None of the given options
before Cricket is equivalent to the number of 36. Direction: What should come in place of the
games played after Badminton. question mark (?) in the following number
29. Which team plays Tambola? Series?
A. Violet B. Yellow 5, 6, 26, 29, 120,?
C. White D. Brown A. 125 B. 1125
E. can not be determined C. 120 D. 64
30. How many teams play between the teams E. None of these
which play Volleyball and Kho-kho? 37. Direction: What will come in place of the
A. None B. 1 question mark (?) in the following number
C. 2 D. 3 series?
E. More than 3 0.8, 3, 7.4, 16.2, 33.8, (?)
31. Which of the following does not belong to a A. 53 B. 69
group? C. 41.8 D. 41.4
A. Long jump B. Volleyball E. 58.4
C. Team Purple D. Team Yellow 38. Direction: What will come in place of the
E. Javelin Throw question mark (?) in the following number
series?

19
www.gradeup.co

655, 440, 316, 253, 227, (?) A. 650 B. 808


A. 205 B. 210 C. 890 D. 900
C. 195 D. 190 E. 760
E. None of these 45. Direction: What approximate value should
39. Direction: What will come in place of the
come in place of the question mark (?) in the
question mark (?) in the following number
series? following equation (Note: You are not
600, 563, 537, 520, 510, ? expected to calculate the exact value)?
A. 500 B. 520 4985.0346 ÷ 215.987 - 3768.112 ÷ 206.868
C. 505 D. 540 +387.021÷42.879-8.99= ?
E. 515 A. 8 B. 5
40. Direction: What will come in place of the C. 18 D. 11
question mark (?) in the following number
E. 15
series?
46. Direction: In these questions two quantities
14, 14, 8, 32, -28, ?
A. 121 B. 213 numbered I and II are given. You have to
C. 92 D. 120 solve both the equations and mark the
E. 240 appropriate option.
41. Directions: What approximate value should Quantity I: 36 men can do a piece of work in
come in place of the question mark (?) in the x days and 48 women in (x + 7) days. The
following question? (Note: You are not ratio of the work done by 5 men to that by 3
expected to calculate the exact value)
women in the same time is 3: 1. Find the
8537.986 - 2416.005 -221.996-
value of x.
342.987×9.82 + 3.99×5.912.978 = ?-
69.876×48.865 + 72.021 × 11.987 Quantity II: A project requires 12 women to
A. 6500 B. 5900 complete it in 16 days. 12 women started
C. 4300 D. 3900 working and after a few days, 4 women left. If
E. 5050 the remaining project was completed in 18
42. Directions: What approximate value should days, in how many days was the whole
come in place of the question mark (?) in the project completed?
following questions? (Note: You are not
A. Quantity II < Quantity I
expected to calculate the exact value)
B. Quantity II > Quantity I
111111.111÷ 1111.111 ÷ 11.111 × 10.1010
× 101.1010 = ? C. Quantity I = Quantity II
A. 11800 B. 8910 D. Relationship can’t be established
C. 11000 D. 9090 E. None of the above
E. 10200 47. Quantity I:A bag contains 4 red, 5 yellow and
43. Directions: What approximate value should 6 green balls. 2 balls are drawn randomly.
come in place of the question mark (?) in the What is the probability that balls drawn
following questions? (You are not expected to
contain exactly 2 green balls?
calculate the exact value).
Quantity II: A box contains 3 yellow balls, 4
5237.897 - 6629.010 + 7153.999 -
2205.102+3452.0218 = ?+862.987×3.899 black balls, 3 white balls and 2 green balls. If
A. 6340 B. 4688 two balls are drawn at random, what is the
C. 5240 D. 3558 probability that both are black?
E. 6290 A. Quantity II < Quantity I
44. Directions: What approximate value should B. Quantity II > Quantity I
come in place of the question mark (?) in the C. Quantity I = Quantity II
following questions? (You are not expected to
D. Relationship can’t be established
calculate the exact value).
E. None of the above
√3422 × √2121 ÷ √1111 × 10 = ?

20
www.gradeup.co

48. In the figure given below, AB ⊥ AD, A. 15m B. 11m


AB ǁ CD, ∠CBD= 28o and ∠BCE= 65o C. 4m D. 2.5m
E. 1m
52. A company made a cuboidal box of sixe
16*12*5 to sell the ice cream, but later on it
was found that the capacity of the box was
14.28% less than the required capacity while
the height of the box was correct, which is 5
inches. As per the requirement he had to
increase the length and breadth of the box in
equal amount. after that percentage increase
in the area of the base of the box is :
A. 12.5 % B. 6.66%
Quantity I:Value of y
C. 16.66% D. 17.25%
Quantity II:Value of x
E. None of these
A. Quantity II < Quantity I
53. There is a price increase on grocery products
B. Quantity II > Quantity I
by 12% and other items by 15%. The ratio of
C. Quantity I = Quantity II
a person A’s expenditure on grocery products
D. Relationship can’t be established
and other items is 3 : 7. Assuming the
E. None of the above
present salary of A as Rs.3570, how much
49. Quantity I. The speed of a boat in still water is
should the salary be increased so that A
15 kmph and the speed of the current is 3
maintain his consumption to the same level.
kmph. It takes a total of 13.5 hours to row
Assume there is no saving and no other
upstream from point A to B and downstream
expenditure?
from point B to A. Total distance (both
A. 4073.37 B. 3057
upstream and downstream) covered by the
C. 874.97 D. 503.37
boat from A to B is
E. None of these
Quantity II. 175 km
54. Even after reducing the marked price of a
A. Quantity II < Quantity I
walk-man by Rs.64, a shopkeeper makes a
B. Quantity II > Quantity I
profit of 15%. If the cost price be Rs.640,
C. Quantity I = Quantity II
what percentage of profit would he have
D. Relationship can’t be established
made if he had sold the walk man at the
E. None of the above
marked price?
50. Quantity I: On selling 17 Toys at Rs. 720,
A. 25% B. 20%
there is a loss equal to the cost price of 5
C. 10% D. 15%
Toys. The cost price of a Toy is:
E. None of these
Quantity II: A boy buys a Toy for Rs. 140
55. Gaurav has a certain amount of money in his
and sells it at a loss of 15%. The selling price
wallet when he goes out. With that money he
is:
can buy 30 apples or 20 oranges. He retains
A. Quantity I > Quantity II
10% of money for paying mobile bill. With the
B. Quantity I ≥ Quantity II
remaining money, he bought some oranges
C. Quantity II > Quantity I
and 15 apples. How many oranges did he
D. Quantity II ≥ Quantity I
buy?
E. Quantity I = Quantity II or Relation cannot
A. 12 B. 10
be established
C. 15 D. 8
51. Two trains P and Q, 100 m long are moving
E. None of these
on parallel tracks at speeds of 20 m/s and 30
m/s respectively. They are travelling in Directions (56-60): 5 mobile phones
opposite direction. The driver of train P sees namely P, Q, R, S and T are available for sale
the driver of train when he is closest to high. on three ecommerce websites Dolphin, Shark
He throws a ball at a speed of 2 m/s which and Whale. The below table gives the
hits the tail of train Q. What is the distance percentage discount offered by the sites on
between the two trains? the phones. Some of the data is missing

21
www.gradeup.co

be increased so that the new ratio becomes 4


: 5 : 3.
A. 3 : 1 B. 1 : 2
C. 4 : 3 D. 3 : 2
E. None of these
62. A, B and C were employed to complete a
work. Initially all of them worked together,
then A left the work and the remaining work
56. The average selling price of phone P across is completed by B and C together. 70% of the
the three sites is Rs 10600. If the phone is work is completed by B and C working
marked up by 25%, what is the cost price of together and total work is completed in 13
the phone? days. Also, A worked with B and C for the
A. Rs 12000 B. Rs 9600 initial 3 days and then A left the work. If C is
C. Rs 10000 D. Rs 10500 1/3rd more efficient than B then find the time
E. Rs 8800 taken to complete the work when, only A and
57. For phone Q, the difference between the B work together.
average discount on Dolphin and Whale and A. 50 days B. 45 days
Whale and Shark is 240. What is the marked C. 50/3 days D. 100/3 days
price of phone Q? E. 60/3 days
A. Rs 8000 B. Rs 12000 63. Ajay, Bharat and Mohan went out for a 100
C. Rs 7200 D. Rs 9600 km journey. Ajay and Mohan started the
E. Cannot be determined journey in Ajay’s car at the rate of 25km/h,
58. The difference between the selling prices of while Bharat walked at 5km/h. After
mobile R on Dolphin and Whale is Rs 300. If sometime, Mohan got off and started walking
the selling price of mobile R on Shark is Rs at the rate of 5 km/h and ajay went back to
6300, what is the discount offered by Shark pick up Bharat. All three reached the
on mobile R? destination simultaneously. What was the
A. 10% B. 12% number of hours required for the trip?
C. 15% D. 16% A. 5 hours B. 8 hours
E. None of the above C. 10 hours D. 6 hours
59. The price of phone S on Dolphin is 10/9 times E. None of these
its price on Whale. The difference between the 64. Out of the 75 students enrolled in the English
prices of phone S on Dolphin and Shark is Rs class, 12% did not qualify to take the final
1080. What is the price of phone S on Whale? examination on account of lack of attendance.
A. Rs 8100 B. Rs 7600 Of those who qualified to take the test, 9.09%
C. Rs 8540 D. Rs 8910 absented themselves from the exam. Two
E. Rs 7290 thirds of those who took the exam passed the
60. Abhijit bought phone T from whale. On examination. 50% of those who passed
learning that the phone was cheaper on secured a first class in the paper. How many
dolphin, he returned the phone and ordered it students who passed the exam scored first
from site Dolphin. Whale charged a return fee class?
of Rs 150 and thus he ended up paying the A. 24 B. 25
same cost as he did while purchasing from C. 20 D. 21
Whale. Now again he returned the phone to E. 22
Dolphin for a return fee of 250 and purchased 65. If the ages of X and Z are added to twice the
it from site Shark. How much effective age of Y, the total becomes 59. If the ages of
discount did he get on the purchase? Y and Z are added to thrice the age of X, the
A. Rs 1600 B. Rs 1800 total becomes 68. And if the age of X is added
C. Rs 2000 D. Rs 2100 to thrice the age of Y and thrice the age of Z,
E. Rs 2400 the total becomes 108. What is the age of X?
61. The number of bananas in three baskets are A. 12 years B. 17 years
in the ratio 4 : 3 : 5. In what ratio, the C. 21 years D. 19 years
number of bananas in first two baskets must E. 11 years

22
www.gradeup.co

A. 4:5 B. 5: 4
Directions (66-70): Answer the questions
C. 4: 3 D. 8: 7
based on the information given below.
E. None of these
The line graphs given below shows the
69. If the owner exchange the marked up percent
discount percent on five different items A, B,
and discount percent offered on item B, then
C, D and E and it also shows the profit/loss
amount (in Rs.) of these 5 items when sold. what is the difference between new and
Note: Positive values indicate the profit original selling price of item B. Also given that
amount while negative values represent loss the total value of discount given on item B is
amount. Rs. 900.
A. Rs. 960 B. Rs. 1260
C. Rs. 1340 D. Rs. 1560
E. None of these
70. Shopkeeper sold items D and E together and
incurred a loss of on item D and

earned profit on E, then cost price of

item E is less than by what amount of the cost


price of item D?
A. Rs. 850 B. Rs. 1050
C. Rs. 1650 D. Rs. 1850
E. Rs. 1350
71. Direction: In this question, two columns I
and II and three sentences are given, which
are divided into two parts. Column I (A, B
and C) consists of first half of each sentence
and Column II (D, E and F) consists of
second half of each sentence. Match column I
66. If marked up percent for item A is 20% on its with column II, so that the sentences formed
cost price and discount amount offered on are both meaningful and grammatically
item D is Rs. 2100, then marked up amount correct. Choose the option as your answer.
of item D is hat percent of discount offered on 71. I.
item A? A) Local officials in 15 areas around the
capital were also asked
B) Volvo cars are understandably superior to
A. B.
their
C) Kohli remains the highest run scorer in
C. D. II.
E. None of these D) was built to protect the land of natives.
67. If the markup in price of item A is Rs. 3000 E) to remove anything in the streets that
and ratio of market price of item A and C is could be used as projectiles.
120: 23, then mark up amount of item C is F) indeed needed but wasn’t given to the poor
approximately what percent of discount
in time.
offered on item A?
A. A-F and C-D B. A-E
A. 34.44% B. 16.67%
C. C-D D. B-D and C-E
C. 8.88% D. 26.33%
E. C-D and B-E
E. 46.67%
72. Direction: In this question, two columns I
68. If there is another item F who’s marked up
amount is 20% more than that of item C and and II and three sentences are given, which
discount rate offered on F is 15%. After are divided into two parts. Column I (A, B
selling item F shopkeeper earns 14.75% and C) consists of first half of each sentence
profit, then what is the ratio of cost price of and Column II (D, E and F) consists of
item C to F if cost price of tem C is Rs. 8000? second half of each sentence. Match column I

23
www.gradeup.co

with column II, so that the sentences formed I.


are both meaningful and grammatically A) Oil prices have plunged 30 percent since
correct. Choose the option as your answer. October as the supply
I. B) Before anyone could do something, Alex
A) Bolt is currently the holder of the world C) The number of road accidents have gone
record in down due to
B) According to several airport operators, II.
C) The government is particularly anxious to D) of everything that is wrong with today’s
increase the society.
II. E) has surged and the global demand growth
D) was Chelsea's plan but it didn’t work as has weakened.
they thought it would. F) among all songs released this year.
E) availability of credit for small and medium- A. A-D B. B-F and C-D
sized businesses. C. A-F and C-D D. C-F
F) Amazon is evaluating the top U.S. airports E. A-E
for new locations. 75. Direction: In this question, two columns I
A. B-F and C-E B. B-D and II and three sentences are given, which
C. B-E and C-F D. A-C are divided into two parts. Column I (A, B
E. B-E and C-D and C) consists of first half of each sentence
73. Direction: In this question, two columns I and Column II (D, E and F) consists of
and II and three sentences are given, which second half of each sentence. Match column I
are divided into two parts. Column I (A, B with column II, so that the sentences formed
and C) consists of first half of each sentence are both meaningful and grammatically
and Column II (D, E and F) consists of correct. Choose the option as your answer.
second half of each sentence. Match column I I.
with column II, so that the sentences formed A) With the supply remaining high, some
are both meaningful and grammatically analysts and market
correct. Choose the option as your answer. B) The match has been moved across
I. continents from Buenos
A) Details of the case against Meng, to be C) There was no sign of life on the rogue
heard in the II.
B) Rohtang pass is one of the highest D) Aires after River Plate fans attacked Boca’s
motorable roads team bus before the original fixture was
C) The President addresses the nation on the postponed two weeks ago.
eve of E) flew a giant inflatable pig painted in Boca’s
II. blue and yellow stripes over the pitch.
D) Christmas lights were installed in the area F) participants say that the cut may not be
only yesterday. sufficient to end oil’s rout.
E) Supreme Court of British Columbia, remain A. A-E and B-D B. A-F and B-D
sparse. C. C-D D. B-F
F) despite batting for the whole day. E. A-D
A. B-D B. C-F
Direction (76-80): In the following passage,
C. A-D and C-F D. B-F and C-D
there are blanks each of which has been
E. A-E
numbered. These numbers correspond to the
74. Direction: In this question, two columns I
question numbers; against each question, five
and II and three sentences are given, which
words have been suggested, one of which fills
are divided into two parts. Column I (A, B
the blanks appropriately.
and C) consists of first half of each sentence
The Syrian Civil War has gone through several
and Column II (D, E and F) consists of
phases over the (###Q1###) of seven
second half of each sentence. Match column I
years and it now appears to be entering
with column II, so that the sentences formed
another one. Government forces have
are both meaningful and grammatically
regained (###Q2###) over much of Syria
correct. Choose the option as your answer.

24
www.gradeup.co

with Russian air support and Iranian ground A. Only A B. Only B


forces. Only Idlib and the territories east of C. Only C D. Only B and C
the Euphrates river remain out of the hands E. None of the above
of President Assad’s (###Q3###). With the 82. Direction: Select the phrase/connector from
U.S. planning an imminent withdrawal from the given options which can be used to form a
Syria, things could soon shift again. single sentence from the two sentences given
Control over Eastern Syria is important for the below, implying the same meaning as
government in Damascus for political and expressed in the statement sentences. Pick
economic reasons. President Assad has on out the option which when used to start a
numerous occasions stated his desire to sentence combines both the above sentences
establish control over the entire Syrian in one.
territory in order to (###Q4###) the image When our channel still had the game of
of a strong and stable regime. Before the war, cricket. I was then interviewing the captain of
Syria produced 387,000 barrels per day of the Indian team and learned a lot about his
which 140,000 bpd were exported. Most of
strategies.
this oil came from Eastern Syria, which is now
A) Four years ago
under the control of the U.S.-backed SDF.
B) Before four years
Currently, the Syrian oil industry is a shadow
C) During four years
of its (###Q5###) self-due to the civil
A. Only A B. Only B
war.
C. Only C D. Only A and B
76. Find the appropriate word in each case.
A. Periodical B. Timeline E. None of the above
C. Course D. Frames 83. Direction: Select the phrase/connector from
E. Age the given options which can be used to form a
77. Find the appropriate word in each case. single sentence from the two sentences given
A. Control B. Energy below, implying the same meaning as
C. Bureaucracy D. Decline expressed in the statement sentences. Pick
E. Delineation out the option which when used to start a
78. Find the appropriate word in each case. sentence combines both the above sentences
A. Personality B. Control in one.
C. Boundaries D. Decision You should bake your cake with patience and
E. Regime interest. You will get a fluffy and delicious
79. Find the appropriate word in each case. cake.
A. Demean B. Dominate A) Till
C. Vile D. Strengthen B) Only if
E. Dwindle C) Despite
80. Find the appropriate word in each case. A. Only A B. Only B
A. Replica B. Former C. Only C D. Only A and C
C. Latter D. Representative E. None of the above
E. Idle 84. Direction: Select the phrase/connector from
81. Direction: Select the phrase/connector from the given options which can be used to form a
the given options which can be used to form a single sentence from the two sentences given
single sentence from the two sentences given below, implying the same meaning as
below, implying the same meaning as expressed in the statement sentences. Pick
expressed in the statement sentences. Pick out the option which when used to start a
out the option which when used in the sentence combines both the above sentences
beginning, combines both the into one. in one.
Poetry lovers tend to be nostalgic by nature. The salon has one of the best hair-stylists in
You would hear a lot from them talking about town. Its services are exclusive.
the declining quality of the audience.
A) Besides
A) Because
B) In addition to
B) Although
C) Along with
C) Since

25
www.gradeup.co

A. Only A B. Only B The President suggested that he


C. Only C D. All A, B and C might __________ to secure the release of
E. None of the above a prominent Chinese
85. Direction: Select the phrase/connector from businesswoman __________ in Canada at
the given options which can be used to form a the request of American authorities.
single sentence from the two sentences given A. transmit, captured
below, implying the same meaning as B. discuss, held
expressed in the statement sentences. Pick C. present, honoured
out the option which when used in D. interact, applauded
the middle of the two sentences, combines E. intervene, arrested
them both into one. 89. Direction: A sentence with two blanks is
Do not forget to pack some extra bottles of given, each blank indicating that something
water and packets of food. There could be has been omitted. Choose the pair of words
scarcity in the area you are going to. that best fit the meaning of the sentence as a
A) Due to whole.
B) As if While there will always be people fearing the
C) In case imminent __________ of either Urdu or
A. Only A B. Only B poetry or both, the flame continues
C. Only C D. Only A and B to __________ in this corner of central
E. None of the above Delhi’s Hazrat Nizamuddin Basti.
86. Direction: A sentence with two blanks is A. renaissance, glimmer
given, each blank indicating that something B. resurrection, blaze
has been omitted. Choose the pair of words C. demise, flicker
that best fit the meaning of the sentence as a D. destruction, attenuate
whole. E. expiration, intensify
Before the Spring Festival ________, we 90. Direction: In the given question, a part of
celebrate the day of cleaning, which is the sentence is printed in bold. Below the
thought to sweep away the bad luck and call sentence, three alternatives to the bold part
for the days of __________. are given which may help improve the
A. ends, destruction sentence. Choose the option that reflects the
B. starts, prosperity correct use of the phrase in the context of the
C. blossoms, calamity sentence. In case the given sentence is
D. kicks, contentment correct, your answer is (E), i.e., ‘No correction
E. encircles, tribulation required’.
87. Direction: A sentence with two blanks is If your new job offer falls under, you can
given, each blank indicating that something come and work for me.
has been omitted. Choose the pair of words A. falls through B. falls in
that best fit the meaning of the sentence as a C. falls down D. falls off
whole. E. No correction required
After the war, commercial radio ______ 91. Direction: In the given question, a part of
began in the 1920s and became an important the sentence is printed in bold. Below the
mass __________ for entertainment and sentence, three alternatives to the bold part
news. are given which may help improve the
A. dispense, channel sentence. Choose the option that reflects the
B. bestow, medium correct use of the phrase in the context of the
C. broadcasting, pedestrian sentence. In case the given sentence is
D. broadcasting, medium correct, your answer is (E), i.e., ‘No correction
E. show, altercation required’.
88. Direction: A sentence with two blanks is She found the curtained casement flung open
given, each blank indicating that something to let in a stream of sunlight and fresh air.
has been omitted. Choose the pair of words A. let up B. let up on
that best fit the meaning of the sentence as a C. let off D. let on
whole. E. No correction required

26
www.gradeup.co

92. Direction: In the given question, a part of Direction (95-100): Read the given passage
the sentence is printed in bold. Below the carefully and answer the questions that
sentence, three alternatives to the bold part follow. Certain words are printed in bold to
are given which may help improve the help you locate them while answering some of
sentence. Choose the option that reflects the these.
correct use of the phrase in the context of the India is technology-savvy to some extent. The
onslaught of technology has hit every sphere
sentence. In case the given sentence is
in our life—we book food, cabs and plumbers
correct, your answer is (E), i.e., ‘No correction
through applications. There are applications
required’. for finding a date, checking into flights before
The hot car seats stung the children's bare you reach an airport, reading the news,
legs and made them cry up in protest. budgeting your income and everything else.
A. cry down B. cry off There have been initiatives to introduce
C. cry out D. cry over technology into the realm of education, but
E. No correction required those have been sporadic and unrelated. The
93. Direction: In the given question, a part of urban population in India is expected to grow
the sentence is printed in bold. Below the faster than its overall population by 2030,
sentence, three alternatives to the bold part according to a report by a leading newspaper.
are given which may help improve the This will put considerable pressure on the
sentence. Choose the option that reflects the economy and education system. Our
correct use of the phrase in the context of the education system currently faces a few
challenges, like the sheer capacity to deliver
sentence. In case the given sentence is
education to all sections of our society, and
correct, your answer is (E), i.e., ‘No correction
the quality of existing institutions.
required’. There is a distinct gap in the education
The couple also plans to educate school landscape. Frost & Sullivan reports that the
children about wildlife conservation quality of higher education is top notch in tier-
and providing an alternate source of 1 universities, but not in tier-3 schools. The
income for hunters and trappers in order to IIT has introduced the National Programme on
stop poaching. Technology Enhanced Learning, a
A. provide an alternative source of income government-funded initiative to help students
B. provide an alternate source of income across the world learn concepts, and provide
C. providing an alternative source of income free access to videos on YouTube. Thus it is
D. will provide an alternate source of income quite clear, that the introduction of
E. No correction required technology is happening, but slowly and only
94. Direction: In the given question, a part of in certain places.
A 2016 Annual States of Education Report
the sentence is printed in bold. Below the
(ASER) survey carried out among 5,60,000
sentence, three alternatives to the bold part
children aged 3-16, in 589 districts in India
are given which may help improve the showed that attendance in primary schools in
sentence. Choose the option that reflects the Uttar Pradesh were as low as 56%, and
correct use of the phrase in the context of the attendance in upper primary schools in Bihar
sentence. In case the given sentence is was 52%. The high dropout rates and low
correct, your answer is (E), i.e., ‘No correction attendance rates are alarming. Will
required’. technology help in making the study-matter
When Benjamin Button starts aging more interesting and engaging to restless
backwards, he sees his loved one's age and students? Given that the majority of India’s
die because he grows younger by the day. population lives in rural and semi-urban
A. his loved one's age and die as areas, it only makes sense if the infusion of
B. his loved ones age and die because technology into education is done on a larger
scale, and at the grass-root level.
C. his loved ones age and die as
Source:
D. his loved ones age and die because of
https://www.thebetterindia.com/122774/digit
E. No correction required
alisation-education-india

27
www.gradeup.co

95. Which of the following has been indicated in 98. Which of the following words is the most
the passage? similar in meaning to "infusion" as used in the
A. Free resources and material are made to passage?
reach through video channels. A. Distortion B. Aggression
B. The urban population is expected to C. Representation D. Combination
increase by 2030 in comparison with the rural E. Adherence
population.
99. With reference to the passage, which of the
C. Quality of education is not a big challenge,
following statements has been mentioned by
but reaching to all the corners of the country
is. Frost and Sullivan?
D. Technology is making education reach at A. The education imparted at the schools is
every corner of the rural India. hugely different from the one imparted at the
E. Attendance in upper primary schools of universities.
Bihar is higher than that in the primary B. There are no defined reasons why the
schools of Uttar Pradesh. attendance of the schools in Bihar is different
96. Which of the following statements is TRUE from that in Uttar Pradesh.
with reference to the given passage? C. There is a lot happening about the fusion of
A. There are applications to book a flight, technology and education at the grassroot
check the status of the flight, and even find level.
the time to deboard.
D. Education is reaching the rural areas by
B. The quality of education in tier-3 schools is
various government-funded initiatives.
lower than that of the tier-1 universities.
E. None of the above.
C. Technology has a big challenge of making
schools accessible to the rural children. 100. Which of the following could be an appropriate
D. Technology has made the access of title to the passage?
education faster. A. Impact of technology on education
E. None of the above. B. Education in 2030
97. Which of the following words is the most C. Quality of education versus its reach in the
opposite in meaning to "sporadic" as used in rural areas
the passage? D. Measures to improve quality of education
A. Abrupt B. Sufficient E. Impact of urban population on the quality
C. Regular D. Innocent of education
E. Occasional

28
www.gradeup.co

29
www.gradeup.co

ANSWERS
1. Ans. D. The codes for all the words are given in the
the code for ‘of’ is ‘go’. following table:
In 1st and 2nd statements, the common word
is ‘need’ and the common code is ‘me’. So,
‘me’ stands for ‘need’.
And, in 2nd and 3rd statements, the common
word is ‘hour’ and the common code is ‘ne’.
The codes for all the words are given in the
following table:

4. Ans. B.
there are 2 empty boxes in the
arrangement.
i) Box which is of Silver colour is placed at
top.
ii) Only white box is placed between Top
2. Ans. E. most box and R.
the code for ‘ge’ can be either ‘must’ or iii) Box T is White colored.
‘have’. iv) There are 5 boxes between P and Silver
In 1st and 2nd statements, the common word colored box.
is ‘need’ and the common code is ‘me’. So, v) S is placed immediately above Brown
‘me’ stands for ‘need’. colored box.
And, in 2nd and 3rd statements, the common vi) Red colored box is placed immediately
word is ‘hour’ and the common code is ‘ne’. below Brown colored, which is P.
The codes for all the words are given in the vii) Number of boxes above P is 1 more than
following table: number of boxes below P.

3. Ans. A.
as the code for:
• pa à honest/people
• ne à hour
Clearly, from the available options “ne so pa”
stands for “good hour people”. (As new word
is needed for “good”).
In 1st and 2nd statements, the common word
is ‘need’ and the common code is ‘me’. So,
‘me’ stands for ‘need’.
And, in 2nd and 3rd statements, the common
viii) Yellow colored box is placed at bottom.
word is ‘hour’ and the common code is ‘ne’.

(1)
www.gradeup.co

ix) Number of boxes between P and Q is


equivalent to that of between R and S.
x) There are more than 7 boxes between Q
and silver colored box.

xvii) Box W is not Red colored.


(so, only possibility is W is silver colored).
xviii) More than 1 empty boxes are there.
xix) One of the box placed is V.
xx) Box Y is yellow colored.
xi) Only Mustard and Magenta colored boxes
are placed between A and P.
xii) A is Pink colored.
xiii) Boxes between T and A are equivalent to
that of boxes between R and U.
xiv) Box U is not placed at top.

5. Ans. E.
either Mustard or Magenta box are placed
between S and A.
i) Box which is of Silver colour is placed at
top.
ii) Only white box is placed between Top
most box and R.
xv) Box Q is green colored. iii) Box T is White colored.
xvi) Box blue is placed somewhere above box iv) There are 5 boxes between P and Silver
Brown. colored box.

(2)
www.gradeup.co

v) S is placed immediately above Brown xiii) Boxes between T and A are equivalent to
colored box. that of boxes between R and U.
vi) Red colored box is placed immediately xiv) Box U is not placed at top.
below Brown colored, which is P.
vii) Number of boxes above P is 1 more than
number of boxes below P.

xv) Box Q is green colored.


xvi) Box blue is placed somewhere above box
Brown.
viii) Yellow colored box is placed at bottom.
ix) Number of boxes between P and Q is
equivalent to that of between R and S.
x) There are more than 7 boxes between Q
and silver colored box.

xvii) Box W is not Red colored.


(so, only possibility is W is silver colored).
xviii) More than 1 empty boxes are there.
xix) One of the box placed is V.
xi) Only Mustard and Magenta colored boxes xx) Box Y is yellow colored.
are placed between A and P.
xii) A is Pink colored.

(3)
www.gradeup.co

viii) Yellow colored box is placed at bottom.


ix) Number of boxes between P and Q is
equivalent to that of between R and S.
x) There are more than 7 boxes between Q
and silver colored box.

6. Ans. E.
box W is placed at top, so no other box can
be placed above it.
i) Box which is of Silver colour is placed at
top.
ii) Only white box is placed between Top
most box and R. xi) Only Mustard and Magenta colored boxes
iii) Box T is White colored. are placed between A and P.
iv) There are 5 boxes between P and Silver xii) A is Pink colored.
colored box. xiii) Boxes between T and A are equivalent to
v) S is placed immediately above Brown that of boxes between R and U.
colored box. xiv) Box U is not placed at top.
vi) Red colored box is placed immediately
below Brown colored, which is P.
vii) Number of boxes above P is 1 more than
number of boxes below P.

xv) Box Q is green colored.


xvi) Box blue is placed somewhere above box
Brown.

(4)
www.gradeup.co

v) S is placed immediately above Brown


colored box.
vi) Red colored box is placed immediately
below Brown colored, which is P.
vii) Number of boxes above P is 1 more than
number of boxes below P.

xvii) Box W is not Red colored.


(so, only possibility is W is silver colored).
xviii) More than 1 empty boxes are there.
xix) One of the box placed is V.
xx) Box Y is yellow colored.

viii) Yellow colored box is placed at bottom.


ix) Number of boxes between P and Q is
equivalent to that of between R and S.
x) There are more than 7 boxes between Q
and silver colored box.

7. Ans. B.
Box V is red colored.
i) Box which is of Silver colour is placed at
top.
ii) Only white box is placed between Top
most box and R.
iii) Box T is White colored.
iv) There are 5 boxes between P and Silver xi) Only Mustard and Magenta colored boxes
colored box. are placed between A and P.

(5)
www.gradeup.co

xii) A is Pink colored.


xiii) Boxes between T and A are equivalent to
that of boxes between R and U.
xiv) Box U is not placed at top.

8. Ans. C.
box R is blue colored.
i) Box which is of Silver colour is placed at
top.
ii) Only white box is placed between Top
xv) Box Q is green colored. most box and R.
xvi) Box blue is placed somewhere above box iii) Box T is White colored.
Brown. iv) There are 5 boxes between P and Silver
colored box.
v) S is placed immediately above Brown
colored box.
vi) Red colored box is placed immediately
below Brown colored, which is P.
vii) Number of boxes above P is 1 more than
number of boxes below P.

xvii) Box W is not Red colored.


(so, only possibility is W is silver colored).
xviii) More than 1 empty boxes are there.
xix) One of the box placed is V.
xx) Box Y is yellow colored.

(6)
www.gradeup.co

viii) Yellow colored box is placed at bottom. xv) Box Q is green colored.
ix) Number of boxes between P and Q is xvi) Box blue is placed somewhere above box
equivalent to that of between R and S. Brown.
x) There are more than 7 boxes between Q
and silver colored box.

xvii) Box W is not Red colored.


(so, only possibility is W is silver colored).
xi) Only Mustard and Magenta colored boxes xviii) More than 1 empty boxes are there.
are placed between A and P. xix) One of the box placed is V.
xii) A is Pink colored. xx) Box Y is yellow colored.
xiii) Boxes between T and A are equivalent to
that of boxes between R and U.
xiv) Box U is not placed at top.

9. Ans. E.
K is either granddaughter or grandson of A
Symbols used are as follows:

(7)
www.gradeup.co

12. Ans. D.
Statement I - Meeta is facing South. Now, if
she turns 90 ° towards her left, she will face
East, which happens to be direction in which
Shilpa is facing.
So, Shilpa is facing East direction.
Hence, Statement I is alone sufficient to
10. Ans. E. answer the question.
K’s gender is not known, so we cannot Statement II - Uday is facing North. If he
predict the exact number of females in the turns 90 ° towards his left, he faces West. It
family. is given that Shilpa is facing in the direction
Symbols used are as follows: opposite to west, that is East.
So, Shilpa is facing East direction.
Hence, Statement II is alone sufficient to
answer the question.
Hence, data either in statement I alone or in
statement II alone are sufficient to answer
the question.
13. Ans. A.
From statement I:
P is mother of M and N.
Q is sister-in-law of N.
Therefore, Q is wife of M.

11. Ans. D.
H is son-in-law of A
Symbols used are as follows:

Thus M is brother of N.
From Statement II:
M and N are children of S.
R is the sister-in-law of M and daughter-in-
law of S.
Therefore, R is the wife of N.

(8)
www.gradeup.co

So, E must have 16 houses.


B > A/F(17) > E(16) > G(15) > D > C(9)
> A/F
Option d) is the correct answer as E has 16
houses.
16. Ans. E.
i) D has more houses than C but less than
G.
Therefore, G > D > C
The sex of M is not clear.
ii) Only 2 persons has more houses than E.
Hence, M is either brother or sister of N.
Therefore, _ > _ > E > _ > _ > _ > _
Hence data in Statement I alone are
iii) The person having second highest
sufficient to answer the question, while the
number of houses has 17 houses.
data in Statement II alone are not sufficient
Therefore, _ > _ (17) > E > _ > _ > _ > _
to answer the question.
iv) Number of houses that G has is multiple
14. Ans. B.
of both 3 and 5. And, it is given that no one
As the colour of clear sky is blue.
has more than 22 houses.
From statement 1: ‘Indigo’ means ‘Grey’,
So, only possibility is G having 15 houses.
‘Grey’ means ‘Black’, Black’ means ‘Blue’
v) C has exactly 9 houses but he does not
Here code for blue is not mentioned.
have least number of houses.
Hence, statement 1 alone is not sufficient.
Therefore, _ > _ (17) > E > G(15) > D >
From statement 2: ‘Black’ means ‘Blue’,
C(9) > _
‘Blue’ means ‘Orange’, ‘ Orange’ means
vi) B donot have 17 houses but has more
‘Green’
houses than A.
Clearly it is given that ‘blue means orange’.
Therefore, we can conclude:
Hence, colour of clear sky is orange.
B > A/F(17) > E > G(15) > D > C(9) > A/F
Thus statement 2 alone is sufficient.
vii) It is also given that no one has equal
15. Ans. D.
number of houses.
i) D has more houses than C but less than
So, E must have 16 houses.
G.
B > A/F(17) > E(16) > G(15) > D > C(9)
Therefore, G > D > C
> A/F
ii) Only 2 persons has more houses than E.
Option e) is the correct answer as either A or
Therefore, _ > _ > E > _ > _ > _ > _
F has least number of houses.
iii) The person having second highest
17. Ans. D.
number of houses has 17 houses.
i) D has more houses than C but less than
Therefore, _ > _ (17) > E > _ > _ > _ > _
G.
iv) Number of houses that G has is multiple
Therefore, G > D > C
of both 3 and 5. And, it is given that no one
ii) Only 2 persons has more houses than E.
has more than 22 houses.
Therefore, _ > _ > E > _ > _ > _ > _
So, only possibility is G having 15 houses.
iii) The person having second highest
v) C has exactly 9 houses but he does not
number of houses has 17 houses.
have least number of houses.
Therefore, _ > _ (17) > E > _ > _ > _ > _
Therefore, _ > _ (17) > E > G(15) > D >
iv) Number of houses that G has is multiple
C(9) > _
of both 3 and 5. And, it is given that no one
vi) B donot have 17 houses but has more
has more than 22 houses.
houses than A.
So, only possibility is G having 15 houses.
Therefore, we can conclude:
v) C has exactly 9 houses but he does not
B > A/F(17) > E > G(15) > D > C(9) > A/F
have least number of houses.
vii) It is also given that no one has equal
number of houses.

(9)
www.gradeup.co

Therefore, _ > _ (17) > E > G(15) > D > i) There are more than 2 persons between Z
C(9) > _ and V, who sits to the left of V.
vi) B donot have 17 houses but has more j) S sits to the right of Y. And it is also given
houses than A. that S and Y are not immediate neighbors of
Therefore, we can conclude: X.
B > A/F(17) > E > G(15) > D > C(9) > A/F
vii) It is also given that no one has equal
number of houses.
So, E must have 16 houses. After filling the remaining data, we get:
B > A/F(17) > E(16) > G(15) > D > C(9)
> A/F
1) Option d) is the correct answer as E has 21. Ans. A.
16 houses. no one sits between Q and W.
2) Option e) is the correct answer as either A a) X sits third from left end of the row.
or F has least number of houses. b) T sits adjacent to X.
3) Option d) is the correct answer as D has c) Z sits to the immediate right of Z.
third lowest number of houses. d) Only 4 persons sit between T and R.
18. Ans. D.
à D > B, not true (as D < A ≤ B).
⇒ E > B, not true (as E = A ≤ B).
Therefore, both the conclusions does not
E. Only 1 person sits between T and Q, who
follow.
does not sit adjacent to X.
19. Ans. E.
f) The number of persons who sit on the right
à 2 ≥ 4, true (as 2 ≥ 3 = 4)
side of Q is 1 more than sitting to the left
à 5 < 2, true (as 2 ≥ 3 = 4 > 5).
side of him.
Therefore, both the conclusions follow.
20. Ans. C.
12 persons are sitting in the row.
a) X sits third from left end of the row.
b) T sits adjacent to X. g) Q sits to the left of W.
c) Z sits to the immediate right of Z. h) Only 4 persons sit between W and O, who
d) Only 4 persons sit between T and R. sits to the right of Q. And, it is also given
that, Only 2 persons sit between W and U.

E. Only 1 person sits between T and Q, who


does not sit adjacent to X.
f) The number of persons who sit on the right i) There are more than 2 persons between Z
side of Q is 1 more than sitting to the left and V, who sits to the left of V.
side of him. j) S sits to the right of Y. And it is also given
that S and Y are not immediate neighbors of
X.
g) Q sits to the left of W.
h) Only 4 persons sit between W and O, who
sits to the right of Q. And, it is also given
that, Only 2 persons sit between W and U. After filling the remaining data, we get:

(10)
www.gradeup.co

22. Ans. A. g) Q sits to the left of W.


as R is sitting third to the left of one who is h) Only 4 persons sit between W and O, who
sitting second to the right of U. sits to the right of Q. And, it is also given
a) X sits third from left end of the row. that, Only 2 persons sit between W and U.
b) T sits adjacent to X.
c) Z sits to the immediate right of Z.
d) Only 4 persons sit between T and R. i) There are more than 2 persons between Z
and V, who sits to the left of V.
j) S sits to the right of Y. And it is also given
E. Only 1 person sits between T and Q, who that S and Y are not immediate neighbors of
does not sit adjacent to X. X.
f) The number of persons who sit on the right
side of Q is 1 more than sitting to the left
side of him.
After filling the remaining data, we get:

g) Q sits to the left of W. 24. Ans. D.


h) Only 4 persons sit between W and O, who a) X sits third from left end of the row.
sits to the right of Q. And, it is also given b) T sits adjacent to X.
that, Only 2 persons sit between W and U. c) Z sits to the immediate right of Z.
d) Only 4 persons sit between T and R.

i) There are more than 2 persons between Z


and V, who sits to the left of V. E. Only 1 person sits between T and Q, who
j) S sits to the right of Y. And it is also given does not sit adjacent to X.
that S and Y are not immediate neighbors of f) The number of persons who sit on the right
X. side of Q is 1 more than sitting to the left
side of him.

After filling the remaining data, we get:


g) Q sits to the left of W.
h) Only 4 persons sit between W and O, who
23. Ans. C. sits to the right of Q. And, it is also given
R sits second to the left of S. that, Only 2 persons sit between W and U.
a) X sits third from left end of the row.
b) T sits adjacent to X.
c) Z sits to the immediate right of Z. i) There are more than 2 persons between Z
d) Only 4 persons sit between T and R. and V, who sits to the left of V.
j) S sits to the right of Y. And it is also given
that S and Y are not immediate neighbors of
E. Only 1 person sits between T and Q, who X.
does not sit adjacent to X.
f) The number of persons who sit on the right
side of Q is 1 more than sitting to the left
After filling the remaining data, we get:
side of him.

(11)
www.gradeup.co

25. Ans. E. Step-III: 18 15 42 78 30 69 48 54 (Multiply


As depicted above, C and I have 5 letters each number by 3)
between them in the given word as well as in Step-IV: 81 51 24 87 3 96 84 45 (Reverse
English alphabet. N and T again have 5 the digits of each number)
letters between them and each of the pairs Step-V: 79 49 22 85 1 94 82 43 (Subtract 2
(S and T) and (N and O) have no letter from each number)
between them. Hence, “79 49 22 85 1 94 82 43” will be the
So, there are 4 such pairs. correct answer.
26. Ans. B. 29. Ans. E.
Logic: Tambola is played by either Brown or Violet
Step I: Multiply the digits of number. team.
Step II: Add 2 in each number. i) Ludo is played in last slot of last day.
Step III: Multiply each number by 3. ii) Team Red plays Cricket on Wednesday in
Step IV: Reverse the digits of each number. third slot.
Step V: Subtract 2 from each number. iii) Javelin Throw is played by team blue
Input: 52 71 43 39 47 36 17 19 immediately before Ludo.
Step I: 10 7 12 27 28 18 7 9
Step II: 12 9 14 29 30 20 9 11
Step III: 36 27 42 87 90 60 27 33
Step IV: 63 72 24 78 9 6 72 33
Step V: 61 70 22 76 7 4 70 31
iv) Number of games played before Cricket is
Hence, “61 70 22 76 7 4 70 31” will be the
equivalent to number of games played after
fifth step of the given input.
Badminton.
27. Ans. D.
v) Number of games played between Table
Logic:
Tennis and Long jump is equivalent to the
Step I: Multiply the digits of number.
number of games played between Badminton
Step II: Add 2 in each number.
and Javelin Throw. Both Table tennis and
Step III: Multiply each number by 3.
Long jump are played before Cricket.
Step IV: Reverse the digits of each number.
Step V: Subtract 2 from each number.
Input: 9 15 7 6 13 22 12
Step I: 9 5 7 6 3 4 2
Step II: 11 7 9 8 5 6 4 vi) Number of teams playing before Long
Step III: 33 21 27 24 15 18 12 jump is equivalent to number of teams
Step IV: 33 12 72 42 51 81 21 playing after Team yellow. And it is also
Hence, fourth step of the input will be “33 12 given that Ludo is not played by team
72 42 51 81 21”. yellow.
28. Ans. E. (so, Long jump can not organized in first slot
Logic: of first day).
Step I: Multiply the digits of the number.
Step II: Add 2 in each number.
Step III: Multiply each number by 3.
Step IV: Reverse the digits of each number.
Step V: Subtract 2 from each number.
vii) Only 3 teams play between team yellow
The first step (Step I) of some input is
and green.
given as - 4 3 12 24 8 21 14 16
(so, we get 2 possible cases here as position
Step I: 4 3 12 24 8 21 14 16
of green is not confirmed).
Step-II: 6 5 14 26 10 23 16 18 (Add 2 in
each number)

(12)
www.gradeup.co

Case 1: xiii) Archery is played immediately before


Tambola.

Case 2:
xiv) Team pink plays kho-kho.
(only possible place is Kho-kho is played on
Sunday).

viii) Team black plays Table tennis.


ix) Team Purple plays Basketball at
12:00pm. xv) Archery is played by neither Brown nor
(so, only possibility is team purple playing on Violet team.
Monday). (so, only possibility is archery is played by
Case 1: orange team).
xvi) Kabaddi is played before volleyball.

Case 2:
After filling the remaining data, we get:

x) Only 2 teams play between Green and 30. Ans. A.


white teams. no team plays between the teams which play
Case 1: Volleyball and Kho-kho.
i) Ludo is played in last slot of last day.
ii) Team Red plays Cricket on Wednesday in
third slot.
iii) Javelin Throw is played by team blue
Case 2: immediately before Ludo.

xi) Team Mustard plays Badminton.


(So, case 2 gets eliminated as it does not iv) Number of games played before Cricket is
satisfy the condition). equivalent to number of games played after
Badminton.
v) Number of games played between Table
Tennis and Long jump is equivalent to the
number of games played between Badminton
xii) Long jump and Tambola are played on and Javelin Throw. Both Table tennis and
the same day. Long jump are played before Cricket.
(so, Tambola must be played on
Wednesday).

(13)
www.gradeup.co

vi) Number of teams playing before Long Case 2:


jump is equivalent to number of teams
playing after Team yellow. And it is also
given that Ludo is not played by team
yellow.
(so, Long jump can not organized in first slot xi) Team Mustard plays Badminton.
of first day). (So, case 2 gets eliminated as it does not
satisfy the condition).

vii) Only 3 teams play between team yellow


xii) Long jump and Tambola are played on
and green.
the same day.
(so, we get 2 possible cases here as position
(so, Tambola must be played on
of green is not confirmed).
Wednesday).
Case 1:
xiii) Archery is played immediately before
Tambola.

Case 2:
xiv) Team pink plays kho-kho.
(only possible place is Kho-kho is played on
Sunday).

viii) Team black plays Table tennis.


ix) Team Purple plays Basketball at
12:00pm.
(so, only possibility is team purple playing on xv) Archery is played by neither Brown nor
Monday). Violet team.
Case 1: (so, only possibility is archery is played by
orange team).
xvi) Kabaddi is played before volleyball.

Case 2:
After filling the remaining data, we get:

x) Only 2 teams play between Green and


white teams. 31. Ans. B.
Case 1: Volleyball is played in different slot other
than those (given in other options). All
others are played in second slot.
i) Ludo is played in last slot of last day.
ii) Team Red plays Cricket on Wednesday in
third slot.

(14)
www.gradeup.co

iii) Javelin Throw is played by team blue Case 1:


immediately before Ludo.

Case 2:
iv) Number of games played before Cricket is
equivalent to number of games played after
Badminton.
v) Number of games played between Table x) Only 2 teams play between Green and
Tennis and Long jump is equivalent to the white teams.
number of games played between Badminton Case 1:
and Javelin Throw. Both Table tennis and
Long jump are played before Cricket.

Case 2:
vi) Number of teams playing before Long
jump is equivalent to number of teams
playing after Team yellow. And it is also
given that Ludo is not played by team
yellow. xi) Team Mustard plays Badminton.
(so, Long jump can not organized in first slot (So, case 2 gets eliminated as it does not
of first day). satisfy the condition).

xii) Long jump and Tambola are played on


vii) Only 3 teams play between team yellow
the same day.
and green.
(so, Tambola must be played on
(so, we get 2 possible cases here as position
Wednesday).
of green is not confirmed).
xiii) Archery is played immediately before
Case 1:
Tambola.

Case 2: xiv) Team pink plays kho-kho.


(only possible place is Kho-kho is played on
Sunday).

viii) Team black plays Table tennis.


ix) Team Purple plays Basketball at
xv) Archery is played by neither Brown nor
12:00pm.
Violet team.
(so, only possibility is team purple playing on
(so, only possibility is archery is played by
Monday).
orange team).

(15)
www.gradeup.co

xvi) Kabaddi is played before volleyball. Case 1:

After filling the remaining data, we get:


Case 2:

32. Ans. A.
i) Ludo is played in last slot of last day.
ii) Team Red plays Cricket on Wednesday in viii) Team black plays Table tennis.
third slot. ix) Team Purple plays Basketball at
iii) Javelin Throw is played by team blue 12:00pm.
immediately before Ludo. (so, only possibility is team purple playing on
Monday).
Case 1:

iv) Number of games played before Cricket is


equivalent to number of games played after Case 2:
Badminton.
v) Number of games played between Table
Tennis and Long jump is equivalent to the
number of games played between Badminton
and Javelin Throw. Both Table tennis and x) Only 2 teams play between Green and
Long jump are played before Cricket. white teams.
Case 1:

vi) Number of teams playing before Long


jump is equivalent to number of teams
Case 2:
playing after Team yellow. And it is also
given that Ludo is not played by team
yellow.
(so, Long jump can not organized in first slot
of first day).
xi) Team Mustard plays Badminton.
(So, case 2 gets eliminated as it does not
satisfy the condition).

vii) Only 3 teams play between team yellow


and green.
(so, we get 2 possible cases here as position xii) Long jump and Tambola are played on
of green is not confirmed). the same day.
(so, Tambola must be played on
Wednesday).

(16)
www.gradeup.co

xiii) Archery is played immediately before jump is equivalent to number of teams


Tambola. playing after Team yellow. And it is also
given that Ludo is not played by team
yellow.
(so, Long jump can not organized in first slot
of first day).
xiv) Team pink plays kho-kho.
(only possible place is Kho-kho is played on
Sunday).

vii) Only 3 teams play between team yellow


and green.
xv) Archery is played by neither Brown nor (so, we get 2 possible cases here as position
Violet team. of green is not confirmed).
(so, only possibility is archery is played by Case 1:
orange team).
xvi) Kabaddi is played before volleyball.

Case 2:
After filling the remaining data, we get:

33. Ans. E. viii) Team black plays Table tennis.


Team pink plays on Sunday at 8:00am. ix) Team Purple plays Basketball at
i) Ludo is played in last slot of last day. 12:00pm.
ii) Team Red plays Cricket on Wednesday in (so, only possibility is team purple playing on
third slot. Monday).
iii) Javelin Throw is played by team blue Case 1:
immediately before Ludo.

Case 2:

iv) Number of games played before Cricket is


equivalent to number of games played after
Badminton.
v) Number of games played between Table x) Only 2 teams play between Green and
Tennis and Long jump is equivalent to the white teams.
number of games played between Badminton Case 1:
and Javelin Throw. Both Table tennis and
Long jump are played before Cricket.

vi) Number of teams playing before Long

(17)
www.gradeup.co

Case 2:

xi) Team Mustard plays Badminton.


(So, case 2 gets eliminated as it does not
satisfy the condition).

35. Ans. E.
W is to the west of K

xii) Long jump and Tambola are played on


the same day.
(so, Tambola must be played on
Wednesday).
xiii) Archery is played immediately before
Tambola.

36. Ans. A.
xiv) Team pink plays kho-kho. The pattern of given series is:
(only possible place is Kho-kho is played on
Sunday).

xv) Archery is played by neither Brown nor


Violet team.
(so, only possibility is archery is played by
orange team).
xvi) Kabaddi is played before volleyball.

37. Ans. B.
After filling the remaining data, we get: The pattern of given series is:
→(0.8 × 2) + 1.4=3,
→(3 × 2) + 1.4=7.4,
→(7.4 × 2) + 1.4=16.2,
→(16.2 × 2) +1.4=33.8,
34. Ans. C.
→(33.8 × 2) + 1.4=?=69,
If she walks for 5m towards west from her
Thus, the missing number is 69
final position (i.e. K), she will reach point W.
38. Ans. E.
The series is,
655 - (63 -1)= 440
440 - (53 -1)=316
316 - (43 -1)= 253

(18)
www.gradeup.co

253 - (33 -1)= 227 46. Ans. B.


227 - (23 -1)= 220 =? Quantity I: Ratio of the efficiency of a man
39. Ans. C.
to that of a woman = =9:5
The pattern of the number is
600 – 563 = 37 = 62 +1 Let the efficiency of one man = m and
563 – 537 = 26 = 52 +1 efficiency of one women be w.
537 – 520 = 17 = 42 +1 5m/3w = 3/1
520 – 510 = 10 = 32 +1 m/w = 9/5 (Effeciency of one man to one
510 - ? = 5 = 22 +1 women)
? = 505 Using the formaula => No. of persons x No.
Hence option C is correct of days x effecieny we get:
40. Ans. C. ⸫ 36 * x * 9 = 48(x + 7) * 5
The pattern of given series is: Or, 324x – 240x = 48 * 5 * 7
→ 14 + (13 -1) = 14, Or, 84x = 48 * 5 * 7
→ 14 - (23 -2) = 8, ⸫ x = 20 days
→ 8 + (33 -3) = 32, Quantity II: LCM of 16 and 18 = 144
→ 32 - (43 -4)= -28, Suppose 12 women do total 144 units in 16
→-28 + (53 -5) =?=92, days.
Thus, the missing number is 92 Then 12 women can do in 1 day=144/16= 9
41. Ans. B. units
8538 - 2416 - 222-3430 + 4×63 = ?- And (12 – 4 =) 8 women do in 1 day
70×49+72×12
= = 6 units
5900 -3430 + 864 = ? -3430 + 864
5900 = ? Work done in last 18 days = 18 × 6 = 108
42. Ans. D. units
Earlier work = 144 – 108 = 36 units, was
done by 12 women in 36/9 = 4 days
⸫ Total time = 18 + 4 = 22 days
Quantity I < Quantity II
47. Ans. A.
Quantity I:
n(S) = 15C2 = 105
43. Ans. D.
n(E) = 6C2 = 15
? ≈ 5238 - 6629 + 7154 - 2205 + 3452 -
863 × 4 ∴ Required probability =
? ≈ 5238 - 6629 + 7154 - 2205 + 3452 -
3452 Quantity II:
? ≈ (5238 + 7154) - (6629 + 2205) n(S) = 12C2 = 66
? ≈ 12392 - 8834 = 3558 n(E) = 4C2 = 6
44. Ans. B. ∴Required probability =
? ≈ 58 x (46/33) x 10
? ≈ 808 Hence, Quantity I > Quantity II
45. Ans. B. 48. Ans. A.
? = 4985.0346 ÷ 215.987 - 3768.112 ÷ Given, AB ǁ CD
206.868 + 387.021÷42.879-8.99 ⸫ ∠ ABC= ∠BCE
? ≈ 4985 ÷216 - 3768 ÷ 207 + 387 ÷ 43 -9 Or, x+28o= 65o
? ≈ 23 - 18 + 9 - 9 = 5 Or, x=37o
From ΔABD,
∠DAB+ ∠ABD+∠BDA= 180o

(19)
www.gradeup.co

Or, 90o+x+y=180o Thus, expenditure on grocery products = Rs.


Or, 90o+37o+y=180o 3*357 = Rs. 1071
Or, y= 53o Expenditure on other items = 7 * 357 = Rs.
Hence, Quantity II < Quantity I 2499
49. Ans. A. New expenditure = 112% of Rs. 1071 +
Upstream speed = 15 – 3 = 12 kmph 115% of Rs. 2499
Downstream speed = 15 + 3 = 18 kmph = 1.12 * 1071 + 1.15 * 2499
Let the distance between A and B = D km = 1199.52 + 2873.85
= 4073.37 = New salary
Now,
Increase in income = New salary – Old
salary
Or,
=4073.37 – 3570
= Rs. 503.37
D= = 36 × 2.7 = 97.2 km 54. Ans. A.
Total distance = 97.2 × 2 = 194.4 km CP of walk man = Rs.640
Quantity I > Quantity II SP of walk man = Rs.640*1.15 = Rs.736
50. Ans. C. MP of walk man = 736 +64 = Rs.800
I. C.P. of 12 Toys = S.P. of 17 Toys = Required % of profit = (800-640)*100/640 =
Rs.720. CP of 1 Toy = 720/12=Rs60. 25%
II. SP =85% of 140 =85/100*140 =Rs119. 55. Ans. D.
51. Ans. C. Let the amount which he has be ‘a’
Since the trains are travelling in opposite Price of 1 apple = a/30
direction velocity for the driver of the faster Price of 1 orange = a/20
train = 50 m/s Distance travelled = length of Gaurav retains 10% of the money.
the train = 100 m Amount remaining = a – 10% of a = 0.9a
Time taken by the ball from one train to the He bought 15 apples.
other Price of 15 apples = 15 × a/30 = 0.5a
= 100/50 = 2 seconds Amount remaining = 0.9a – 0.5a = 0.4a
Ball in thrown at 2 m/s,
distance between the two trains=2x2 = 4 m. Number of oranges bought = =8
52. Ans. C. 56. Ans. B.
Original volume = 16*12*5 = 960 inch3 Suppose the marked price of phone P is
Now this Volume is 14.28%(i.e., 1/7) less 100P
than the the Required volume. Then its selling price on the sites are 85P,
Let Required Capacity be 'x' ⇒ 960 92P and 88P respectively.
= (1-1/7)x ⇒ x = 1120 inch3 (85P + 92P + 88P)/3 = 10600
Now, Base area in the Required capacity box => P = 10600 x 3/265 = 120
= 1120/5 = 224 inch2 Hence MP of P = Rs 12000
& Base area of given cuboidal box = 16*12 = CP = 12000/1.25 = Rs 9600
192 inch2 57. Ans. A.
So, Increase in area = 224 -192 = 32 inch2 Suppose the marked price of phone Q is
∴ % increase = (32/192) *100 = 16.66% 100M and the discount offered by Whale on it
53. Ans. D. is N%.
Let the expenditure on grocery products and Discount on Dolphin = 19M
other items be 3x and 7x respectively Discount on Whale = MN
So, 3x + 7x = 3570 Discount on Shark = 13M
10x = 3570 According to the given condition,
x = 357 (19M + MN)/2 – (13M + MN)/2 = 240

(20)
www.gradeup.co

3M = 240 Increase in number of bananas in 2nd basket


=> 100M = 8000 = 5b - 9b/5 = 16b/5
58. Ans. D. Ratio in which number of bananas is
Suppose the marked price of mobile R is increased = 8b/5 : 16b/5 = 1 : 2
100R 62. Ans. C.
SP on Dolphin = 76R Let, the total work be 100 units
SP on Whale = 72R Number of units completed by B and C
76R – 72R = 4R = 300 working together = 70 units
=> 100R = 7500 Number of units completed by A, B and C
Discount offered by Shark = 7500 – 6300 = working together = 100 – 70 = 30 units
1200 Since A, B and C together worked for only 3
Discount % = 1200/7500 x 100 = 16% days
59. Ans. E. Number of units completed by A, B and C
Suppose the marked price of mobile S is working together = 30/3=10 units
100S Time taken to complete the whole work = 13
Price on Whale = 81S days
Price on Dolphin = 81S x 10/9 = 90S So, number of days B and C work together to
Price on Shark = 78S complete the remaining work = 13 – 3 = 10
90S – 78S = 1080 days
=> 12S = 1080 So, number of units completed by B and C
=> 100S = 9000 work together in one day = 70/10=7 units
=> 81S = 90 x 81 = Rs 7290 So, number of units completed by A alone in
60. Ans. C. one day = 10 – 7 = 3 units
Suppose the marked price of mobile T is Since C is 1/3rd more efficient than that of B
100T So, number of units completed by C alone in
Price on whale = 89T one day = 4 units
Price on dolphin = 88T So, number of units completed by B and A
Given, 88T + 150 = 89T work together in one day = 6 units
=> MP of phone T = Rs 15000 Time taken by A and B to complete the work
Price on shark = 15000 x 0.84 = Rs 12600 = 100/6=50/3 days
Total price paid = 12600 + 150 + 250 = Rs So option (c) is the correct answer.
13000 63. Ans. B.
Effective discount = 15000 – 13000 = Rs Let us consider the total distance from A to D
2000 is = x + y + z
61. Ans. B. Where AB = x
Let the number of bananas initially in the BC = y
three baskets be 4a, 3a and 5a respectively. CD = z
Let the number of bananas finally in the
three baskets be 4b, 5b and 3b respectively.
Given, no change in number of bananas in Since the time taken by Ajay to cover x + y
3rd basket is done. + y at the speed of 25km/h is same as the
Thus, 5a = 3b time taken by Bharat to cover x at the speed
a = 3b/5 of 5 km/h, therefore
Now, number of bananas in 1st basket (x + y + y)/25 = x/5
initially = 4 × 3b/5 = 12b/5 y = 2x
Number of bananas in 2nd basket initially = 3 similarly, the time taken by Ajay to cover y +
× 3b/5 = 9b/5 y + z at the speed of 25km/h is same as the
Increase in number of bananas in 1st basket time taken by Mohan to cover z at speed of
= 4b - 12b/5 = 8b/5 5km/h, therefore

(21)
www.gradeup.co

(y + y + z)/25 = z/5
y = 2z
so, we have y = 2x = 2z So option (a) is the correct answer.
x: y: z = 1: 2: 1 67. Ans. B.
it implies that x = 25 km, y = 50 km, z = 25 Let cost price of item A = 100x
km Marked price of item A = 100x + 3000
thus, the required time = (x + y + y + y + Selling price of item A
z)/25 = 200/25 = 8 hours. =
Hence, (B) is the correct option.
64. Ans. C. Cost price of item A
out of 75 students, 12% did not qualify for =
final 15x=1800; x = 120
Remaining = 75-12 % of 75 = 66 Discount offered on item A
Also 9.09% out of 66 were absent =
Present = 66 – 66 × 1/11 = 60
Marked price of C
Passed = 2/3 × 60 = 40
No. of students who got 1st class = 40 × =
50% = 20
65. Ans. A. Cost price of C
X + 2Y + Z = 59 …. (!) =
3X + Y + Z = 68 ….. (!!) Marked up amount of C = 2875 – 2500 = Rs.
X + 3Y + 3Z = 108 … (!!!) 375
Solving these equations together X = 12 Required percent
years, Y = 15 years, Z = 17 years
Aliter =
Multiplying equation (ii) with 3, we get So option (b) is the correct answer.
9X + 3Y +3Z = 204 68. Ans. C.
Now subtracting equation (iii) from bove Let CP of C = Rs. 8000
equation Let MP of C = 100x
We get, SP of C = 80% of 100x = 80x
8X = 96 ⇒ x=12 CP of C = 80x + 200 = 8000
66. Ans. A. x = 97.5
Let CP of A = 100x Marked up amount of C = 100x – 8000
MP of A = 12% of 100x = 120x = Rs. 1750
SP of A = 85% of 120x = 102x Marked up amount of F = 120% of 1750 =
CP of A = 102x – 750 = 100x Rs. 2100
x = 375 Let CP of item F = 100y
Discount amount offered on A = 120x – 102x MP of F = 100y + 2100
= 18x = Rs. 6750 SP of F = 85% of (100y + 2100) = 114.75%
Let MP of D = 100y of 100y
SP of D = 76% of 100y = 76y 85y + 1785 = 114.75y
100y – 76y = 2100 y = 60
y = 87.5 CP of F = 100y = Rs. 6000
CP of D = 76y + 350 = Rs. 7000 Required ratio = 8000: 6000 = 4: 3
MP of D = 100y = Rs. 8750 So option (c) is the correct answer.
Marked up amount = 8750 – 7000 = Rs. 69. Ans. D.
1750 Let the marked price of item B = 100x
Required percent = SP of B = 88% of 100x = 88x

(22)
www.gradeup.co

Discount amount = 100x – 88x = 900 72. Ans. A.


x = 75 B states what several airport operators have
Cost price of B = 88x – 600 = Rs. 6000 to say. F explains what is being said. Thus,
MP of B = 100x = Rs. 7500 B-F forms a pair. C talks about the
SP of B = 88x = Rs. 6600 government being anxious about something.
Marked up percent E explains the issue which is the availability
of credit for small and medium-sized
=
business. Rest of the options don’t make
After interchange of marked up percent and sense when paired together. The correct
discount percent answer is option A
MP of B = 112% of 6000 = Rs. 6720 73. Ans. E.
SP of B = 75% of 6720 = Rs. 5040 'A' introduces the case against Meng, which
Required difference = 6600 – 5040 = Rs. is further explained by E. 'E'informs the
1560 location where the case will be presented.
So option (d) is the correct answer. Rest of the options don’t make sense when
70. Ans. E. paired with each other. The correct answer is
Let marked price of item D and E is ‘100x’ option E.
and’100y’ respectively 74. Ans. E.
SP of item D = 76% of 100x = 76x 'A' talks about oil prices falling by 30 per
CP of item D = 76x + 350 cent and E states the reason for the same:
Loss incurred on item D "Increase in the supply and the fall in the
global demand growth". None of the other
=
two segments can be joined to convey a
logical sense.
75. Ans. B.
'A' talks about the supply of oil remaining
high. F explains what the analysts and
152x + 700 = 23500
market participants said. B and D
x = 150
connect logically stating that the match
CP of item D = 76x + 350 = Rs. 11750
between Boca Juniors and River Plate was
SP of item E = 90% of 100y = 90y
moved to another continent due to the
CP of item E = 90y – 400
violence that took place earlier among the
Loss incurred on item E
fans of the two football clubs. Hence the
= correct answer is option B.
76. Ans. C.
The word ‘course’ means a phase. Since the
y = 120 phrase after the blank indicates the time of
CP of item E = 90y – 400 = Rs. 10400 ‘seven years’, ‘course’ would be an
Required difference = 11750 – 10400 = Rs. appropriate representation of it. ‘Course of
1350 seven years’ would be meaningfully
So option (e) is the correct answer. appropriate. ‘Age’ does not represent a
71. Ans. B. period, therefore, stand incorrect.
'A' states that the local officials were to do ‘Timeline’ is also an incorrect word to fill the
asked something. E explains what they were blank as it defines a planned period of
asked to do- 'remove anything in the streets specific events. ‘Frames’ is plural and will not
that could be used as projectiles'. Rest of the fit n the blank.
options don’t make sense when paired "Periodical" refers to two things:
together. The correct answer is option B. i. a magazine or newspaper published at
regular intervals. [Noun]

(23)
www.gradeup.co

ii. occurring or appearing at intervals; replicate its replica. ‘Latter’ is also not
occasional. [Adjective] correct since latter or something coming
Both of these meanings will not make after is not known in this case. Therefore,
appropriate sense in the given blank. something which has not happened cannot
77. Ans. A. be copied. ‘Representative’ does not fit
The hint to fill the blank lies with the use of grammatically and ‘idle’ means something
‘regained’ and ‘with the support of ground inactive and is irrelevant in the context. The
forces’. At this point, we know that it should word ‘former’ is the most appropriate word to
be a positive word. From the given options, fill in the blank since the former, or the one
we may, therefore, remove ‘decline’ previously existing can be copied.
(meaning decrease) and ‘delineation’ 81. Ans. C.
(meaning the act of describing or portraying Both sentences are logically related to each
something). other. Therefore, ‘although’ would be ruled-
The words ‘energy’ and ‘bureaucracy’ would out as it is used to connect contradictory
not give a meaningful completion to the statements. ‘Because’ would also get
sentence since ‘regaining energy’ or cancelled as we do not start a sentence with
‘regaining bureaucracy’ over something does the word.
not require intervention from the ground The correct sentence would be: ‘Since poetry
forces. So, the correct word is "control". lovers tend to be nostalgic by nature, you
78. Ans. E. would hear a lot from them talking about the
The word that should fit in the blank must declining quality of the audience’.
refer to President Assad and relate with his 82. Ans. A.
governance. This way it would make the The sentence initiates with the narrator
meaning complete. From the given options, stating that they still had the game of cricket
the only words that fall in this category are followed by an interview of the captain of the
‘boundaries’ and ‘regime’. The word Indian team. From the given options, ‘before
‘boundaries’ would not be correct since four years’ is grammatically incorrect.
‘President Assad’s boundaries’ would literally ‘During four years’ would not be appropriate
mean his limitations. Therefore, ‘regime’, to use since it is a very wide timeframe and
which means a form of government, would cannot precisely place the event of
make the best fit. interviewing the captain of the Indian team.
President Assad’s ‘personality’ or ‘decision’ The word ‘four years ago’ would connect both
would not relate to the former part of the the sentences appropriately as: ‘Four years
sentence. ago, when our channel still had the game of
79. Ans. D. cricket, I was interviewing the captain of the
We need a word in the blank which shows Indian team and learned a lot about his
what the desire (control over the entire strategies’.
Syrian territory) of the President aims at 83. Ans. B.
(formation of the image of a strong and The second sentence is the outcome of the
stable regime). Logically speaking, the first sentence, therefore, using the word ‘till’
establishment of control over the territory would be inappropriate. Therefore, option A
will lead to the establishment of the image of would be cancelled.
a strong and stable regime. Thus Similarly, the word ‘despite’ would be used in
"strengthen" is the most appropriate word. the cases where the end result is
The other words are completely irrelevant. contradictory. ‘Only if’ would meaningfully
80. Ans. B. connect the action and the result perfectly.
The sentence mentions that the Syrian oil The correct sentence would be: ‘Only if you
industry is a shadow of something. ‘Replica’ bake it with patience and interest, you will
cannot be correct as nothing needs to get a fluffy and delicious cake.

(24)
www.gradeup.co

84. Ans. D. which is to sweep away the bad luck and call
Both the given sentences are similar in for the days of prosperity.’
nature and characterize the good services of 87. Ans. D.
a salon. Therefore, all three options would Looking at the phrase ‘commercial radio’
combine both the sentences correctly. followed by ‘began in 1920s’ helps us
The first sentence would be: ‘Besides having understand that there is a need for a word
one of the best hair-stylists in town, its that is related to the radio. From the given
services in the salon are exclusive’. options, we may therefore, shortlist
The second sentence would be: ‘In addition ‘broadcasting’ and ‘show’. The other words
to the salon having one of the best hair- mean differently and do not convey an
stylist in town, its services are exclusive’. appropriate meaning when placed in the first
The third would be: ‘Along with having one of blank.
the best hair-stylist in town, the services of ‘Bestow’ means presenting or offering
the salon are exclusive’. something and ‘dispense’ means distributing.
85. Ans. C. None of these could be used to represent the
The connector ‘as if’ is used to represent information transmitted by a radio. At this
something that seems from known or given point, we have options C, D and E
information. The phrase ‘due to’ is used to shortlisted.
connect action and reaction’. In the given Now, we must look at the second part of the
sentences, we see that the first sentence is a sentence. The second half of the sentence
desired action while the second one is the helps us understand that it is about radio
reason for it. being an important ‘something’ for giving
In the given conditions, ‘in case’ would be information to the masses. Therefore, the
the perfect connector. The sentence would word ‘medium’ fits as the only option.
become: ‘Do not forget to pack some extra The complete sentence would be: ‘After the
bottles of water and packets of food in case war, commercial radio broadcasting began in
of scarcity in the area you are going’. the 1920s and became an important mass
86. Ans. B. medium for entertainment and news.’
The hint to finding the correct answer lies in 88. Ans. E.
looking at the second blank first. From the The hint to the correct answer lies in the
second half of the sentence, we may second half the sentence, where we read ‘the
understand that it is about the people doing release of a prominent Chinese
certain things to sweep away the bad luck businesswoman’. This helps us in
and calling for the days of something, for understanding that the word that must fit in
which we need to fill the blank. Since the the second blank must be related to her
people are trying to get away from bad luck, arrest or captivity. Therefore, options A, B
the second blank would be a positive word. and E are shortlisted.
This helps us in cancelling options A, C and E Now, looking at the first part of the sentence,
as ‘destruction’, ‘calamity’ (disaster) and we may understand that the president can
‘tribulation’ (trouble) are all negative words. only ‘discuss’ when another participant is
From the remaining options, we may now mentioned in the sentence, which in this case
look at the first set of words. From ‘starts’ is not. ‘Transmit’ means to pass something
and ‘kicks’, the former would be a better to someone, which does not give a meaning
choice to fill in the blank since ‘kicks’ cannot here since we see no mention of anything
be an appropriate word to be used with being passed on.
spring festival. The correct sentence would be: ‘The
The sentence would be: ‘Before the Spring president suggested that he might intervene
Festival starts, there is a day of cleaning, to secure the release of a prominent Chinese

(25)
www.gradeup.co

businesswoman arrested in Canada at the 91. Ans. E.


request of American authorities.’ To let someone or something in
89. Ans. C. means to allow them to enter.
To fill in the first blank, we must take the If bad weather or an unpleasant
hint from the phrase ‘people fearing the situation ‘lets up’, it stops or improves.
imminent’ with respect to Urdu and poetry. ‘Let up on’ someone/something means to
We may guess that people would not fear stop treating someone severely, or
any sort of positive act related to the to stop doing something so forcefully.
language or an art form. The word must ‘Let somebody off’ means
therefore, be a negative word related to the to not punish someone who
decline. At this point, we may hence, has committed a crime or done
shortlist the words ‘demise’, ‘destruction’ and something wrong, or to
‘expiration’. The words ‘renaissance’ and not punish them severely.
‘resurrection’ get dismissed due to their ‘Let someone on’
meanings related to rebirth. means to tell other people about something
We now know that something people feared that you know, especially when it is
to die still ‘continues’, as the second half of a secret.
the sentence highlights. From the remaining The sentence is correct as the sunlight enter
words, we may cancel out ‘attenuate’, which the room when the curtains are opened.
means reduction of effect. Hence, the correct answer is E.
From options C and E, we may decline option 92. Ans. C.
E as the word ‘expiration’ is used for To ‘cry up’ means to praise publicly in order
something having a fixed period, while to enhance in value or repute.
neither Urdu nor poetry has a fixed period. When something is ‘cried off’, it means that it
The correct sentence would be: ‘While there was abandoned or aborted.
will always be people fearing the imminent To ‘cry out’ means to speak in a loud voice,
demise of either Urdu or poetry or both, the or to say something loudly or from a
flame continues to flicker in this corner of distance.
central Delhi’s Hazrat Nizamuddin Basti.’ ‘Cry over’ isn’t a phrasal verb, but rather a
90. Ans. A. part of the idiom 'cry over split milk', which
‘Fall under’ somebody's influence/spell means to express regret about something
means to be affected by someone in a strong that has already happened or cannot be
and often negative way. changed.
If an arrangement, plan, or deal ‘falls In the context of the passage. when the
through’, it fails to happen. children felt the heat, they screamed or cried
If a roof or ceiling ‘falls in’, it collapses and out. Hence, the correct answer is option C.
falls to the ground. 93. Ans. A.
If an argument, organization, or person ‘falls Option A has no mistakes.
down’ on a particular point, they are weak or To 'alternate' is to take turns, while an
unsatisfactory on that point. 'alternative' is an option. The hunters and
If something ‘falls off’, it separates from the trappers don’t plan to come back to
thing to which it was attached and moves poaching. So, there is no question of taking
towards the ground. turns. Rather, hunters and trappers must be
The speaker is providing an alternate job provided other livelihood options, thus,
option if the first option doesn’t work out. 'alternative’ is the correct word.
Thus, with reference to the context of the The ‘to’ infinitive before educate must also
passage, option A is the correct answer. appy to the verb provide in order to maintain
parralelism (using elements in sentences that
are grammatically similar or identical in

(26)
www.gradeup.co

structure, sound, meaning, or meter). So, ‘Abrupt’ means something happening


“provide” should be in its base form. suddenly, ‘sufficient’ means adequate,
Hence, the correct answer is A. ‘innocent’ means not guilty, and ‘occasional’
94. Ans. C. means something happening irregularly.
Option C has no mistakes. Except ‘occasional’, which is the exact similar
The loved ones ageing and dying and of ‘sporadic’, none of the other words relates
Benjamin's de-ageing are two parallel to the word. Therefore, option C, ‘regular’ is
activities. They didn’t die because of the correct answer.
Benjamin's de-ageing. So, the correct word 98. Ans. D.
choice is 'as'. Also, "one's" (contraction of The word ‘infusion’ means amalgamation or
“one is”) shows possession, which is not union of two things. Below are the meanings
present in the sentence. It represents the of the words given as options.
people loved by Benjamin, hence the plural - Distortion: the act of disturbing something
(ones) should be used. The correct answer is - Aggression: Act of anger
option C. - Representation: Presentation on the behalf
95. Ans. A. of self or someone
The second paragraph of the passage - Combination: Union of two or more
mention that several government-funded elements
initiatives are taken to produce free - Adherence: Sticking to something
education sources and spread through From the above, we may clearly derive that
Youtube. ‘combination’ is the closest in meaning to
Apart from this, none of the other options ‘infusion’.
has been mentioned anywhere. 99. Ans. E.
96. Ans. B. The second paragraph clearly states that
The statement mentioned in option B is Frost and Sullivan mentioned about the gap
clearly given in the second paragraph. The between the quality of education between
line stated: ‘Frost & Sullivan reports that the the tier-1 universities and tier-3 schools.
quality of higher education is top notch in None of the given options mentions about
tier-1 universities, but not in tier-3 this. The options do include various things
schools.’ The other options have not been mentioned in the passage, but none has
indicated in the passage, however, the been reported by Frost and Sullivan.
similar statements given in the passage 100. Ans. A.
might sound confusing to the reader. Unless Except A, all the options highlight a specific
the direct information is given in the section of the passage. However, option A is
passage, the statement must be understood the only one giving the big picture of the
as false. entire passage, further confirming that this
97. Ans. C. passage is about the infusion of the
The word ‘sporadic’ represents something technology and education.
irregular or happening at uneven intervals.
Therefore, the nearest opposite would be
‘regular’.

***

(27)

S-ar putea să vă placă și